Você está na página 1de 625
Al: JEE ADVANCED (UJEE) +17 Yes (2002-2018) JEE MAIN (AIEEE) MATHEMATICS Coverage of 100% JEE Main & Advanced Questions GUARANTEED 4 disha Years (1978 - 2018) JEE ADVANCED (IIT-JEE) + 17 Yrs (2002 - 2018) JEE MAIN (AIEEE) MATHEMATICS Topic-wise Solved Papers Aligned on Class 11 & 12 Syllabus Must for JEE Main, Gaited Mamta Batra Advanced, BITSAT & FEMSTicom Other State Engg. Exams MEZEDES * Corp. Office : 45, 2nd Floor, Maharishi Dayanand Marg, Corner Market, Malviya Nagar, New Delhi-110017 Tel. : 49842349) 49842350 Ele) & ‘Typeset by Disha DTP Team DISHA PUBLICATION ALL RIGHTS RESERVED © Reserved 'No part ofthis publication may be reproduced in any form without prior permission of the publisher. The author and the publisher do not take any legal responsibilty for any errors or misrepresentations that might have crept in. We have tried ‘and made our best efforts to provide accurate up-to-date information inthis book. For further information about the books from DISHA, Log on to www.dishapublication.com or email to info@ dishapublication.com GP_3480 10. il. 12. 1B. 14. 15. 16. 17. 18. 19. 20. 21. 22. Beer POC tg Ba ue JEE Advanced 2017 Solved Paper Ba ac CHAPTERS BASED ON CLASS 11' SYLLABUS Trigonometric Functions & Equations ‘Complex Numbers Quadratic Equation and Inequations (Inequalities) Permutations and Combinations Mathematical Induction and Binomial Theorem ‘Sequences and Series Straight Lines and Pair of Straight Lines Girdle Conic Sections CHAPTERS BASED ON CLASS 12 SYLLABUS Functions Limits, Continuity and Differentiability Differentiation Properties of Triangle Inverse Trigonometric Functions Matrices and Determinants Applications of Derivatives Indefinite Integrals Definite Integrals and Applications of Integrals Differential Equations Vector Algebra and Three Dimensional Geometry Probability Miscellaneous (Sets, Relations, Statistics & Mathematical Reasoning) 1-6 7-14 15-20 21-24 25-28 29-34 35-40 41-48 49-58 59-64 65-74 75-78 79-84 85-88 89-98 99 - 108 109 - 112 113 - 126 127 - 132 133 - 152 153 - 162 163 - 170 BP_3480 Solutions 6sExplanations 1. Trigonometric Functions & Equations 1-16 2. Complex Numbers 17-32 3. Quadratic Equation and Inequations (Inequalities) 33-49 4, Permutations and Combinations 50 - 56 5. Mathematical Induction and Binomial Theorem. 57-70 6. Sequences and Series 71-85 7. _ Straight Lines and Pair of Straight Lines 86 - 105, 8. Girde 106 - 127 9. — ConicSections 128 - 154 10. Functions 155 - 167 11. Limits, Continuity and Differentiabilty 168 - 193 12. Differentiation 194 - 201 13. Properties of Triangle 202 - 220 14, Inverse Trigonometric Functions 221 - 225 15. Matrices and Determinants 226 - 242 16. Applications of Derivatives 243 - 268 17. Indefinite Integrals 269 - 276 18. Definite Integrals and Applications of Integrals 277-316 19. Differential Equations 317-327 20. Vector Algebra and Three Dimensional Geometry 328 - 365 21. Probability 366 - 382 22. Miscellaneous (Sets, Relations, Statistics & Mathematical Reasoning) 383 - 394 * The chapters have been divided as per the Class 11° & 12" syllabus followed by the NCERT books. Some of the chapters which are split in the class 11% & 12 syllabus in NCERT have been combined. There might be certain topics/ chapters which are not covered in NCERT but are a part of JEE Advanced/IIT-JEE syllabus. ES 3 er te) PAPER -1 ‘This section contains 6 questions. Each question has four options (4),(B), (Cand (D). ONE or MORE THAN ONE ofthese four options is (are) correct. For a non-zero complex number 2, let arg) denote the principal argument with ~x--nx}, defined by (t)= arg(-1+ it) for all ¢¢ R , is continuous at all point of R, where j= Ji (© Forany wonon-zero complex numbers 2, and 2» )-satep ante) isan integer muliple of (©) Forany three given distinct complex numbers 2,23 and =, the locus ofthe point zsatisf¥ing the condition = tesonaseieting Tnatriangle PQR, let ZPQR = 30° and he sides PQ and QR hhave lengths 10V3 and 10, respectively. Then, which of the following statements) is (are) TRUE? (A) ZQPR=45° (B)TheareaofthetrianglePQRis 25/3 and ZQRP= 120° (© Theradiusoftheincrcle ofthe triangle POR is 103-15 (D) The area ofthe circumeircle ofthe triangle PQR is 10x Let R:2x+y—2=3 and Ph:x+2y-+2=2betwoplanes, ‘Then, which ofthe following statement(s)is (are) TRUE? (A) The line of intersection of P, and P, has direction ratios 1,2,~1 Bx-4_1-3y {-2, 2} with (£0)? +f)? =85, which of the following statement(s) is (are) TRUE? (A) Thereenist r,s €R , where -R and g:R—R be two non-constant differentiable functions. It se and f()= 802 statement (sis (ae) TRUE? (A) fQ)<1-tog,2 © a()>1-Iog,2 eF0V-£0)y¢(x) forall eR, » then which of the following B) fQ)>1-loge2 (©) g()<1-l0g,2 6 Let f-{0,2)->R bea continuous function such that Se nee [fe'rom forall x ¢[0,20). Then, which ofthe following statements) is (are) TRUE? (A) The curve y= f(x) passes through the point (I, 2) (B) Thecurve y= f(x) passes through the point (2,1) (© Thearea ofthe region {(soyet0urR:foysysvi-w} is (D) Thearea ofthe region foxmetorR:soysysvi-r} is Fares cee esi eee as oe ereerecneye reece ae see era at aca omer ‘the second decimal place; ¢.g. 6.25, 7.00,—-0.33,—30, 30.27,—127.30) 1 1 1. Thevalueot (1og,9)2) 3089) (JF) 847 8 Themumber ofS digit numbers which are divisible by 4, with digits from the st (1,2, 3,4, 5} and the repetition of digits is allowed, is 9. Let X be the set consisting of the first 2018 terms of the arthmetie progression 1,6, 11, ., and Y be the set consisting ofthe first 2018 terms of thearthmetic progression 9, 16,23, Then, the number ofelements inthe set XUY is_ 10, The number of real solutions ofthe equation sin” (ee Da (Here, the inverse trigonometric functions sin~'x and cos"! assume valuesin [ z5| and [0, x], respectively) JEE Advanced 2018 Solved Paper 11, Foreach postive integer et Yn =H + 1Min +2). ny" For xR, let [x] be the greatest integer less than or equal tox. If lim y, =, then the value of (2) is 12, Let @ and & be two unit vectors such that @-5 =0. For some x,y €R, let 2 = xa+yb-+(@xB).If|é|=2 andthe vector ¢ is inclined atthe same angle «to both dand 6, then the value of 8 cos? a is. 13. Let a, b € be three non-zero real numbers such that the has two equation acon tinx=ene[- Ez 4 distinct real roots a and B with «+=. Then, the value of bis 14, A firmer F has land in the shape of triangle wit vertices at P(0,0), Q(, 1) and RQ, 0)-From this land, aneighbouring farmer F, takes away the region which les between the side PQanda curve ofthe form y=,"(n>1). Ifthe area ofthe region taken away by the farmer Fis exactly 30% of the area of APQR, then the value of mis Ths eto sms 2 ung, seo ct pH ‘there are 2 questions. Each question has four options (A), (B), (C) Simnonvaeadmeaeteel ee Let S be the circle in the xy-plane defined by the equation Bayles, point Pg(l, 1) and parallel to the x-axis and the y-axis, eerie aetmaemerarme aie oe B) 4) +G-4P=16 © (e-4)y-4)=4 ote GP_3480 MATHEMATICS 16. Let P bea point onthe circle S with both coordinates being, positive. Let the tangent to at P intersect the coordinate axesat the points Mand N. Then, the mid-point ofthe line segment MN must lie on the curve (224 y% e202 (A) (e+) =3y ©) Pyar? © x+y?=2y PARAGRAPH-A ‘There ae five students S, Sp, $3, S, and Sin a music class and for them there are five seats Ry, Ry, Ry, Ry and R, arranged in a ‘ow, where initially the seat Ris allotted tothe student), 2, 3,4, 5. But, on the examination day, te five students arerandomly alloted the five seats 17. 18. as ‘The probability that, on examination day, the student Sy ‘gets the previously allotted seat Rj, and NONE of the remaining students gets the seat previously allotted to him/ heris 3 1 WO a ® 5 7 1 © Os For i= 1,2,3,4, et 7, denote the event that the students S, and §,,,; do NOT sit adjacent to each other on the day of the examination. Then, the probability of the event TaRaRaT is a © et © i 1 5 a4 JEE Advanced 2018 Solved Paper PAPER -2 Soe ‘This section contains 6 questions. Each question has four options (A),(B), (C)and (D). ONE or MORE THAN ONE ofthese four ‘options is are) correct. 1. For any positive integer n, define, : (0,0) > Ras fa) @)= Dt tert oral $O= Lian 3 Terpasy) x€(0,x9) Here, theinverse trigonometric function tan vats ‘Then, which ofthe following statements (are) TRUE? (A) Dja1 ta? GO=55 DP 4G O) see? GO)=10 (©) Forany xed positive integer, tim tan f(x))= 2 (©) Forany fixed postveintegern, tim s<"(f (0) 2. Let The the line passing through the points P{-2, 7) and 002-5), LetF, be the set ofall pairs ofcicles(S,5,) such that T's tangent to, at P and tangent to S at and also such that S, and S, touch each other ata point, say, M. Let E, be the set representing the locus of M asthe pair (S, S:) varies F, Lette set ofall straight line segments joining pair of distinet points of, and passing through the point R(, 1) be F, Let B, be te set of the mid-points of the line segments in the set F,. Then, which of the following statements) is (are) TRUE? (A). Thepoint (-2, 7) lies in E, (tps (£2) senor ent © Thepoint (4) 1) liesinE ©) tpn (03) sesnor eins by 3. Let Sbe the set of all column matrices | b, | such that 5, by +, bs, € Rand the system of equations (in real variables) bas atleast one solution. Then, which of the fllowing system(s) (in real variables) has (have) at leat one solution 4 foreach |b | € 5? bs wy by dy +52 byand x +2y +62 by ® S29 +62= by and2—y—32=b, © 5 2e—dy + 105 by ands—2y+ Se=by (O) x#2y452=by,2e+ 32 by and xt dy—Se= by ‘Consider two straight lines, each of which is tangent to ‘both the circle x2 + y?= ; and the parabola y? = 4x. Let these lines intersect at the point Q. Consider the ellipse whose center is atthe origin © (0,0) and whose semi-major axis is OQ. Ifthe length ofthe minor axis ofthis ellipse is V2 ,then which ofthe following statements) is (are) TRUE? 1 (A) For the ellipse, the eccentricity is 1 and the length, “ iP vis i ofthe latus rectum is 1 (B) Forthe lise, the coventricityis andthe length of the latus rectum is 4 (© Thearea of he region bounded by the ellipse between 1 the linesx sade Lis e(-2) (D) The area of the region bounded by the ellipse between the lines x 1 1 Se andx= lis (@-2) ee 65 2) Let , tbe non-zer0 complex numbers and L be the set of solutions = =x +iy (x, y, €R, i= V=T) ofthe equation sctiZ +r =0, where 7 =x~ iy. Then, which of the following statement(s)is (are) TRUE? (A) IfL has exactly one element, them jsf (B) Iffs=|e,then Las infinitely many elements (©) The number ofelements in Ln {2 :Je-1+i]=5} isat smost2 (D) IfL has more than one element, then L has infinitely ‘many elements GP_3480 MATHEMATICS 6 Let/:(0,x) > Rbea twice differentiable function such that L(x) sint~ f(sinx lim sin? x for all € (0,2). ra It 1(3)- then which ofthe folowing statements) is(are) TRUE? wy AF oA) ® se) (© Thereexists a © (0.m) such that /"(a)=0 aD So forallx e(0.8 a (2) © WZ Scraesanan value reuse oe nee er ai fe ae aloes ‘the second decimal place; ¢ g. 6.25, 7.00, -0.33,—.30, 30.27,-127.30) (HP d-)4 8. Let Pbea matrix oforder3 «3 such thatall he entries in P| ae from the set {-1, 0, 1}. Then, the maximum possible value ofthe determinant of Pi, 9. Let X'bea set with exactly 5 elements and Y be a set with exactly 7 elements. Ifa isthe number of one-one functions from Xto Yand fis the numberof onto functions from to Kothen the value or (Bais 10, Let/:R-> R bea differentiable function with f(0)= _y= fe) satisfies the differential equation 7 # as Fe Ot SSY-2), then the value of im (a) is Il, Let: RR bea differentiable function with f(0) =I and. satisfying the equation Set: WIL O) +S LO) forallx,yeR Then, the value oflog, (/(4) is, 12, Let Pea point in the fist octant, whose image O in the planes-+y-= 3 (thatis the line segment PO is perpendicular to the plane x + y = 3 and the mid-point of PQ lies in the plane x+y=3) lies on the z-axis, Let the distance of P from the ‘canis be . IR is the image of P inthe xy-plane, then the length of PR is AS 13. Consider the cube inthe first octant with sides OP, OQ and OR of length 1, along the x-axis, y-axis and z-axis, spectively where 00, 0,0) thea, Let 2 be the centre ofthe cube and T be the vertex of the cube ‘opposite tothe origin O such that S lies onthe diagonal OT. SR and ¢=ST , then the value of MBx a) x (Fx7)| is_ 14. Let X= (MC P+ IOC + 319)? +» + 1000%C yg), where 1°, {1,2,-, 10} denote binomial coefficients ‘Then, the value of — x 1430 ine TistadUt'h tarenever eecmeeenet Sec aeestea ier ennierk outed Pee ee 1s, Letgy~ freRnetani >} wit [stim ()}nemtnme (Here, the inverse trigonometric funetion sin 4) assumes valuesin| F- bi Lets S(s)=log, (4) and g: E, > Re the finetion defined by ne (=(5)) 1) > R be the function defined by P.Therange off is 1 Q Therange of geontains 2. R_Thedomain offeontains 3. S. Thedomain of gis 4 [ r| 9,040.29) a6 16. FOR Vn. oP OF ‘The correct option is (A) P44, 092; Rot; Sor B) P43, 043, R46, 855 © P34, Q52, R41; 836 @) P54, 093, R46 S55 In ahigh school, «committee has tobe formed from a group OF 6 boys My, My, My, Ma, Mg, Mg and 5 gitls Gy, GG, GG Let a, be the total number of ways in which the ‘committe can be formed such tha the committeghas 5 members, having exactly 3 boys and? girl (i) Let ay be the total number of ways in which the ‘committe can be formed such that the committee has at least 2 members, and having an equal number of boys and gins (i) Let a be the total number of ways in which the ‘committe can be formed such that the committee has ‘5 members, at least 2 of them being girls (iv) Let a be the total number of ways in which the ‘committe can be formed such that the committee has 4 members, having atleast 2 girls and such that both -M; and G; are NOT in the committe together. List-1 List The value of is 1 136 ‘The value ofa is 2 19 ‘The value ofa is 3 1m ‘The value of is 4 20 5 31 6 46 ‘The correct option is (A) P44; 046, R42; S51 B) Pl; Q4; R92; $3 © P+4; 046, R45; $2 ) P94; Q52; R43; S31 ey ae _o-plane whose conjugate axis LM subtendsan angle of 60° atone ofits vertices N. Let the area ofthe triangle LAAN be Let H , where a> b> 0, bea hyperbola inthe a3. LIsT-1 List-t ‘Thelength ofthe conjugateaxisofHis 1. 8 4 ‘The eccentricity of His 7 * B 2 ‘The distance betwoen the foci of H is 3 ‘The length of the latus rectum of His 44 er JEE Advanced 2018 Solved Paper ‘The correct option is: (A) P94; Q2; Rol; $3 @B) P4; Q>3, Rol S92 © P4; Q>1; R93; S52 @) P53; Q4; R52; S51 Leafs ROR sn: 0st © hein for?) [sin] tan? 1 ifx=0 itxe0 where the inverse @ hO= trigonometric function tan“! x assumes values in (34) Gi) F, (2) [sin log, (x+2))], where, for R,[f] denotes the greatest integer less than or equal tof, Hosin(t) itxeo Ce 0 ifx=0 ust-1 ust-m Thefinctonjis ‘1. NOTeotinuousatx=0 Thefuncionyis 2. continuousatx= Oand NOT dfeniabietx=0 Thefinctionjis —3._-1; R52; s (© P>4,Q52;, RoI; s @) P>2; Qo1; R94, $3 GP_3480 MATHEMATICS ar SOLU Paper - 1 (A,B, D) ) maci-9 == (Wyisfatse x-tan'9,120 tan Mt<0 B= arg(-1+i0, [ lim fe 10" LHL #RHL = fis discontinuous at -= 0 (B)isfalse. x and lim f()=m © sno 2) es sae ps = 2+ args; —argzy arg +argz) 2, multiple of 2x + (C)istrue (== )@)~=3) ) (2-23). -4)), » of (e-aX-2) > Gae=a) a ) (2 = ) 3) lay 7, are coneyelic. ie. z lies onacirle (Dyis false. GD) P PQ? +QR?-PR? 13 2PQ-0R (A) c0s30" 5 B_ant orate? 27 axats10 => PR?= 100 or PR=10 ZPaZQ-30 *. (A)is false. (© Araoraron= PQH ORs? l prov x10x Also Z R= 180°-30°-30°= 120° @)istrve © r=4 259 25V3___Sv3. s (ages) 10+5V3 243 2 =sV5@-3)=10V3 -15 ~ (Cpistrue. abe _10V3%10%10 pa She INS 1010 10 ©) Raa 4x25 8 ‘Area ofircumeircle= aR? = 100% ~ (D) iste, cD) (A) Direction ratios of line of intersection of two planes willbe given by 85 g(a@=0 > 2f(a)f (a) +2 (a)E"(a)=0 > 2@ita+"@)=0 If F(a) =0= g(a) =[f(a)?? and [f(a]? <4 g(a) 285 (is not possible.) Hence f(a) +"(a)=0 for ae (xj x2) €(-44) (Distr 8 BO Given £'(x) = "9-0. g(x) GP_3480 JEE Advanced 2018 Solved Paper =o APE) =F BR) Integrating both sides, we get eH) gH 46 a eH 4 AO og D 478 gD) gH) 2) But given that ( el pe oe pel > ef) gett 2 e et 2 ang rtd 2 => -f2) f(2)>1-In2_— and g(l)>1-In2 (B) and (C) are True BO fogataee [Zeina D> Alxy=t-axte8 [Fe'n(nyae 2 t= 2808 fetoavete 001 > ()-2109) tsa linea differential equation, Fae Solution: £(x)xe"* = fe x-3)dx f(x) =-x-+1 +00"* From definition of function, 0) =1 Clearly curve y= 1 ~x, does not pass through (1, 2) but it passes through (2,~ 1) MATHEMATICS (A)is falseand (B) is true Also the area of the region I-xsysVi-x? ,isshown in the figure, is given by op = Area ofquadrant Area AOAB. = emt? Late = 2 4 2 4 (©) istrueand (D) is false. 1 8) (Clog, 9)? #292) (fT) 0847 1 eta 4 = (log, 9)" 80929? 47 4 qlee? = (lo, 9)%0 ers (625) The last 2 digits, in 5~ 16+(n~1)35 < 10086 => 35n-19< 10086 1010s => ns i005 999.7 388 n=288 (XU) =m(X) +n(¥)=n0K0 ¥) 2018 +2018-288=3748 @ 9 oy Mxtxt +x) x xG242x-1) ore 3 M02 + 2x-IK4 =x?) 41 22—3x-K?Y =O > xfx) +2x7 +5x-2]=0 = x= orn + 282+ 5x2: = kx) (say) vecaanent ¢ One rootofp(x) = Oties in (0. ) Thao so ete Ean (oH ns? ne3 nen)" > log = fleet +x =faioet =I te = log2—[x—log 1+, =log2=1+og2=2l0g2-1 negt-inenie tm @) Given | & makes angle o with both & and 6 Also, a+ ybeaxb cosa => x=2cosa. cosa => y=2e0sa a= |cose+ (eosapb+axi a0 JEE Advanced 2018 Solved Paper 3 OP adewtatdeosar faxi? 1 w 02g |= 1x1xsin90° > Aepowtast (© 18x = = Beota=3 13. (0.5) Given that the equation x aI 4 lcaor* a I has two roots a and B, such that +B = 5 V3 acosa+2bsin @ and J3 cos B+2bsin B= Equation of Ey:y=1 ad 5 aces + 2bsinB o) Equation of FF :x=1 subtracting exuation 2) fom (1) we get cn (eos -cos )+26(sin a—sin)=0 Bysymmetrytangensat and willmeet — Faasin Bn 2B 4 3 2ean ig! on y-axis and tangents atF and F; will meet = “ie 2 2 22 2 2 ‘on x-axis B= (5.1) &F,=(1.8) Equation oftangent at Ey: JSx+y=4 => -W3a sin2+4beos™ 3: Git tbeosE=0 Equation oftangent at F,: x+y =4 Points E5(0,4) and Fy(4,0) Lip > -NBax5 +46 0 = betios Tangents at G, and G, arex = 2 and: a2 interseating each other at GQ, 2. Clearly, Fs and G, icon thecurvex + y=4 4) 16 (D) Letpoint Phe (20s ,2 sin) ‘Tangent at P: x cos 0+ y sin 8= 2 a For locus of mid point (x,y) of MN, Shaded area = 32 Ar(APQR) ws a . > [eendiabon _ @ mf 3 17. (A)_ No.of dearrangements for 4 students 143 =12-44 oh . Las : + required probability GP_3480 MATHEMATICS 18. (©) Total cases=5!= 120 Favourable cases Shecease=14 Mot i200 Paper - 2 “required probability 1 (ABAD) fC —— 1 (A,B,D) fy6 aim) = Sar [x2 (x+i +) TE OF DE ED, = Stones j)-tan-Has j-0 > £(0)=tan (+n) tan! (x) = and f,(0)=tan-"(n) . tan®(tan-tn é e «6x (A) San? (G0) = D7? =O = 55 i i 1 RO= 1-1 3 +O Sara} So aira2 see?(fy(0))=se0?(tan"'(n))=1+n? —— see" (fq(0)) > 1+) 3 aa => (1+f,(0))-see(f, (0) =1 w w Sag oysa*e(oy=S4-10 (©) tim tan(f,(x))= lim | —* © nso tn ta ©) tim see? (fy (x) = lim 1+ tan? (fy60)=1+ li tan? (Fy (0))=1 M locus of Misa circle having PQ as its diameter andis given as By 266-2142) + (~TH¥+5)=Oand x42 and itscentreis(0, 1) ‘Locus of B (midpoint) isa cirele having RC sits diameter and is given as. Eq:x(x=1) +(y-1P =0 "Now, alr checkingall he options, we get (D) isthe correct option. (As. D) Here A= 050 fr atleast one solution, we have =A, =Ay=0 }> common tangent P (eadical 25 =>b,+7by =13bs 0 (ay aro ‘The equations have unique solution Option (A)is correct (avo ‘The equations have unique solution option (D) is correct (a=o0 = equations are x—2y+5; », xodytS2n 8h ys x-2y+S2=by ‘The planes given in option (c) ate parallel so they must be coincident aa i) Equation (i) satisfies equation (i) forall by, b, by Option (C) is correct ray frag @a-|5 2 6 l2 1-3) Hi 3 Also A,=|2 2 bs=0 ra For infinite solutions, A, and A, must be 0 1b 3 = |s b& 12» - => b, +b, + 3b; =O which does not satisfy (i) fr all by, by, », so option (B) is incorrect 4 AO Let unin sonmen gat: ym 1 oor Vie? | V2 => mitm-2=0 = m=dl Equation of common tangents are yoxsland y=—x-1 Q=41.9) Equation of ellipse is, GP_3480 JE Advanced 2018 Solved Paper © y aH T0* 1 Las -Roquredarea = 2.f Jpvinwax “l 4l GD) sevider=0 o Tee ‘Adding (i) and (i), we get (HDEHGHDzH=0 ‘Subtracting i) from (i), we get (HZHS-Dzr(-N=0 — Q) Equation (1) and (2) represent set of lines. For equation (1) and (2) to have unique solution, we have: t45 547 (On solving the above equation we get Itisl Option (A) iscorrect For equation (1) and (2) to have infinitely many solutions, and TT +sr-st=sr+st-tr-TF > tr (Tr Hisiirl > ItlrHlsiirl > ItHs| If|t] =|, lines will be parallel for sure but may not be coincident (ie., does not have infinitely many solutions). (©) : Locus of Z isa null set or singleton set ora line, in all, threse cases it wll intersect given circle at most two points. (D): im this case locus of.isa line so L has infinte elements ee = 6 (BCD) tim £€x)sint=f(t)sinx 2. } | tim LODEOSt=F(Osinx _ 2 sax [a+x? a-»°]* > x (Using Hospital's Rule) (+N3)dx => f(x)cosx—f"(x)simx =sin?x. ay ap ay 8 Suppose P=|by 2 by 12 8 Ja, a2 a3 So,det(P)=|bj by bs 2 4 1 (bye bye) — 89 yes —ye) +a (ye bye) +Svncia) Maximum valuecan be 6 when a, = 1 and bye, = byes = bye, = 1 bye, = bye, So, (3) (bs) (0,¢2)==1 and_ (be) (bse) (by6,)= | Therefore jbybye,¢ has two values 1 and —1 which is not posible Contradiction also occurs ifa,= 1," 1,451 and bye2 = bey = For maximum value tobe S one ofthe terms should be zero but this will make 2 terms zero therefore answer should no be 5 fx) there exist « € (0, x) for which f{a)=0 (D) Here, (x) =—2eosx-+-xsinx Hence maximum value ofthe determinant of P=4, aad 9. (19) Heten(X)=5 and n(¥) Number of one-one function = a=7C, «5! and Number of onto function ¥ to X is given as: ~ /® \ \e/ \/ LLLL3 11122 SE ste x 51-0043 x7C)51 Basra “5 Gag atx=0,; 7 5f(x)-2 20 wtin |S = 5 lim f)-2=0 = Tim f= 2. @ GP_3480 JE Advanced 2018 Solved Paper Fix+y)= £0) O)+L OEY) o After putting x= y=0, we get fO=2F OO) => HOF £0)=1) Now putting y=0 in equation (1), we get F(a)= F(a) £0) +1"(R) £0) @ = log, f)= 5 + logge 2 f@)=e%? = f(y) (f=) > bate X= agity=2 Suppose coordinates ofP are (a, ,). So, coordinates of@ ae (0, 0,<) and coordinates of R are(a,b,-2) Here, PQis perpendicular tothe planex + y=3 So, PQs parallel tothe normal of given plane ic. (i+ bis parallel (+) > ab ‘As mid-point of PQ lies in the plane x + y=3,s0 Sarb=68 a 2 ee (given) Therefore, distance of from thex-ax > y 20.1.0) fra, 1) 00,0, Pao. © 0.0.1) MATHEMATICS Here, p Hence, (6 x) x @ xi) a a [Gx Gx Gx dl= f= 14. (646) Srey any ree Cnr Cr mi mc, , So, (NCP +2, BUMS +. A10(NC oF 10 = Ex)? 10% = X dw, Hence, 7430" 143, C= 646 18, (A) For E> 0 and x41 xe(—=,0) U0) xT 7. ®) Aas For Ey, ate( 2 ern > Fx) € (-=,0) (0,20) (oe) Lab Here, a committe ha tobe formed form a group of Boys and 5 girs ‘Total number of ways for selecting exactly 3 boys and 2girls=*C, «C= 20 = 10=200 “Total number of ways for sclectng at east 2 members with equal numberof boys and girls (Cx 5Cy) + (Cy x 5Ca) + (C4 % 5C3) + OC, CQ) + (6C, *5C.)=MC,—1=461 =a, ‘Total numberof ways for selecting 5 members having Cy =8Cy—6C, «C=C, 81= B(x) Total numberof ways for selecting 4 memebrs having at least girls M, and G, are not selected together (M; selected & G; not selected) +n(G; selected & M, not selected) + n(M, and G, both not selected) = (FC, «50, +4C,)+ 10, «50, +40, «°C, +4Cy) HEC, HC, x50, +4C,5C,) 189-04 =34e4tel Aa ‘Area of ALMN= 443 (given) 2 bet Mnon The eccentricity of H ‘The distance between the foci of H= 2ae 2 aden Fe and length of latus ractum of H -mlontaat | which does not exit. 3b > a=2N3 @ wa GP_3480 JE Advanced 2018 Solved Paper So for (P), (2) is eoreet. sim (0)= im [se] = tan'x xd safes soo BD “tan in| ra tim [ tim Ps Se ean which does not exist, so for Q, (1) is correct. in £8) = fm sino (+2) if x 50=>(x+2)-92 = loge (x +2) log, 2<1 = 0< lim singlogg(x +2) < sin Jim sinloge(x+2) lim|sindogs (+29) =0 fy(x)=0 Vxe[-Le*?-2) Pya)=0 Yee e™?—2) "y@)=0 Yxe(-1e?—2) Sor (R), (4) iscomect. y= Hine) =o sn al aaa cost+2xsin4, x20 £'4(0)= lim 4(0)= im, £40 1 Jim cos x0 x sim 46) = im sos + 2x sint} which does not exist So for(S), (3) is correct. 6. ‘The integral 5 sin? xcos?x jin x08" 8 __ay sequal to (Gin cooK sin? x +i costae cost? +c © Theotx +1 1 —1 4c Teo x © Sra (tere Cis a constant of integration) ‘Tangent are draw to the hyperbola 4x2 —y2 =36 atthe points P and Q.Ifthese tangents intersect atthe point (0, 3) then the area (in 9, units) of APTQis. () s4¥3_ @) ov3 G8) 36V5 Wass ‘Tangent and normal are drawn at P(16, 16) on the parabola y? =I6x, which intesecttheaxisoftheparabolaatA and, respectively. IfCis the centre ofthecircle through the points P,Aand Band ZCPB=0,,thena valucof tan is 8 4 1 M2 @3 On > Let i bea vector coplanar with the vectors 43}-£ and b=}+k. IF § is perpendicular to and i-B~24, then [a? is equal 0 (ly 31s 2) 256 @) & 4) 336 If @BeC are the distinct roots, of the equation x2—x+1=0, then a2! 4 p!07 is equal to: Oo @1 @2 1 Let 4(2)=cosx?,f(0)=Vk. and «, B(@- 1) is Mo @1 @2 @- JEE MAIN 2018 - MATHEMATICS POC ROE PAU et APA) 10, n n 2B. 4. 2 .y bein AR.such that Yay.) 416 a and a5 +343 =66. Ifa? +03 +...+09; =140m, then mis equal O8 OH OB 6 . A If Y(xj-5)=9 and Dei =45, then the standard deviation ofthe 9 items xX... X48 aw 4 2 @B 3 ao ‘PQRis a triangular park with PQ=PR= 200m. AT.V. tower stands at the mid-point of QR. Ifthe angles of elevation of the top ofthe tower at P. Qand Rare respectively 45°, 30° and 30, then the height of the tower (in is wo» @) 100y3 @) soy2 4) 100 ‘wo sets Aand B arc asunder A= {(a,b)€RXR:Ja—S|<1 and|b-S] <1}; B= {(a,b)ERxR:4(a~6)? +9(0~5)? $36}. Then © Acs @) AMB=6 (an empty ser) @) neither AC Bnor BoA @ Bea From 6 diferent novels nd; diffrent dictionaries, 4 novels and | dictionary ae tobe selected and arranged in a row on a shelf so thatthe dictionary is always in the middle. The ‘number of such arrangementsis: (D lessthan 500 (@) at east 500 but less than 750 G)_ atleast 750 but less than 1000 @) atleast 1000 10,1} 1¢ £2 tnen the loel minimum value of x) is 2) © -3 @ wi Ow 3 For each tER, let [t] be the greatest integer less than or equal tot. Then sol EE) (1) is equal to 15. @)_ doesnot exist (in R), ame 16. n. 1». 20. a. 2. OF Of OF OF A bag contains 4 red and 6 black balls. A ball is drawn at random from the bag, its colour is observed and this ball along with two additional balls of the same colour are re- turned tothe bag, Ifnow a ball is drawn at random from the bag, then the probability that this drawn ball is red, is, 1 3 3 5 Ms OF OH Thelength ofthe projection ofthe line segment joining the points (5,—1,4)and (4, —1, 3) on the plane, x+y +: a? af of oF If sum of all the solutions of the equation 2 MF a x.) eos {oo E+x} of - } 1 in (0, x] is kx, then kis equal to B 8 20 2 oF oF oF wF A straight the through a fixed point (2, 3) intersects the coordinate axes at distinc points PandQ. If isthe origin and the rectangle OPRQ is completed, then thelocus of Ris () 2x+3y=3y @) 3x+2y=xy @) 3xt2y=6xy —) Bx42y=6 ‘Let Abe the sum ofthe first 20 terms and B bethe sum ofthe first 40 terms ofthe series 422432424? 4524262 If B=2A = 100%, then Ais equal to: () #8 @) 6G) HH MD Ifthe curves y? = 6x,9x? + by? =16 intersect eachother at right angles, then the valu of bis 7 9 My @4 Oz as Letthe orthocentre and centroid ofa trianglebe AC-3, S)and BG, 3) respectively IfCis the circumcentre ofthis triangle, then the radius ofthe circle having line segment AC as diam. eis is NS ® wid @ af a ow m4 25, 2. 29. 30. GP_3480 JEE MAIN 2018 Solved Paper Let $ = {teR:f(x)=|x-n|(-1)sin|x| is not differentiable at}. Then theset Sis equa to © 0 @ G) 0.x} 4) O(anempryset) k-4 2x 2x Mx x4 2x |=(A+Bxyx— A)? then the ordered bx ax xd pair(A, B)isequal to @ 4,3) @ 4,5) @) 45) 4) C45) She tan csr ~~ (Pv qv (~ pq) is equivalent to @ Pp @Qq B~a ap Maden ir qos tas nan snositn 2h Eng on o-» oO» 0 Let S={xeR:x20 and 2|Vx-3] +WVx(vx -6)+6=0. Then. () contains exactly one element. 2) contains exactly two elements G) contains exactly four elements. @ isancmpty set If the tangent at (I, 7) to the curve x? = y~6 touches the circle x? + y? +16x+12y-+e=0 then the value of cis BS QS BH @) 195 Let y~y(x) be the solution of the differential equation dy sinx 5+ yeos rem. 1 (20, ne (5) vena a © 387 @ 4 Osa” intersection of planes x+2y-2-3=0, 1 1 1 1 O58 O35 OF OFF MATHEMATICS SOLUTIONS sina [Gin eos? nysin x rood nF sin? xeos? x pe = pte sex 4 sin} x +0085 ae Crna? Now, put (1+ tan'x)=t = Biante seekk dx 1 tpt dy + ae Gast) Stope of (m,)= + : Slope of PB (a) =-2 2 (4) Here equation of hyperbola is 7 ‘Now, PQ is the chord of contant Hence, tan =feem -| ‘mm 0. ¥Q). Equation of Qis e en = tun 0=2 4 @ 8,8 & Bare coplanar y - - (axb)xa = 24a? b-(a.b)a} =A Se toi =A-i+2}eaky (0, 3) x’ x Q) @, Bareroots ofx?—x + IR and B=-0 ls where «is cube root of unity 1 6 Arenof aor =< TRPQ PaQS.-12) % TR=3+12=15, Arenof aPQT = 1x15x6y5 =455 593 units Also, gos) =cosx t) uommanszunis one - amuae Pat ane e @ GP_3480 JEE MAIN 2018 Solved Paper ‘Since we know that, 10.) (ray +(x-a)> Pp = 2ECyx HC Xa HC yet] (Veal +6-Vea) =28Cyx5+5C, x'09— HC AGE 1? 2Ex+ 10x — 1038+ 5x7 10x4+ Sx) ‘Sum of coefficients of odd degree terms= 2. B 2 M 7 13 : Vag =416 = 3r0,+484]=416 Letheight oftower MN=h & 2 TnAQMN wehave a,+24d=32 a) Now, a) +,,=66=> 2a, +50d=66 .Q) Fromea. (1) & 2) we get; d= 1 anda, =8 uu ‘Also, Ya? = S{8+ (DIP =140 m a) aoa ny Q) Seen? 40m InAPMQ weave: a MP= (200)? = (3? u YP +14r+49)=140m From (2), weget a ¥(200)? (3h)? = hh =100m 1838), | (1718 _ (PEED eo ES seman 0 ed) A=(00)eRXR:a-SKLB-SkH m= Leta-5,=x,b-S=y 2 Set A contains al points inside | x|< Lye Given Y(xi-5)=9 Pix) =54 6) B= (,b)ERXR:a~6) + 9B-S}?<36}, ‘Set B contains all points inside or on ° Also, (4) 57 = 45 a y? 2, 8 BrP -10Ps,r9eH)=45 6) From (ian y (41-1) liesinside the ellipse. Hence, ACB. 12, (4) + Required number of ways= 6c, x3) x4! ‘Standard deviation = VVarianee = 2 = 153 *24= 1080 MarueMarics 6. 1. Q) Here, hoo wen x-b 00 4 oda? im Rth-n|(e™"—Dsin| w+h|-0 re 1 Now take xe, then > 0 12 sku 221 B=B «ordered pair (A, B)is (4, S) ~@vav(-pra) (-pa~q)v(-pag) =-pat-qva) =pate-p For non zero solution ofthe system of linear equa tions; Pk 3 bk -2=0 4 3] Skeil Now equations become xe lly+32=0 ) Bx My=2 2) 2x+4y—32=0 -B) ‘Adding equations (1) & (3) we get 3x + 15y=0 = x=-Sy Now putx =-Sy in equation (1), we get Case-l: x [0,9] 28-Va)+x-6/E +6=0 ax Wk 412-094-429 Since x €[0,9] Case: x €[9,->] 2k -3)4x-6/E +6=0 a x-AVe= 059%, 6.0 Since x [9,9] Hence, x=4 8 16 Equation of tangent at (1, 7)ox?=y—6is 2x-y +50. Dx-yt5-0 GP_3480 iss JEE MAIN 2018 Solved Paper "Now, perpendicular fom centre O(-8, -6)t0 3 2x-y+ 5=0 shouldbe equal toradius ofthe circle ‘Then, ysinx = 2x2—% isthe solution 7 z > W5=Vi00=¢ 0-95 29.) Consider the given differential equation the 30. (1). Equation of plane passing through the line of sinxdy + yeosedx =4xdx interseation of frst wo planesis = dtysine)= 4rd ee sides (Qx-2y432-2)+Aa-yezt=0 = ysmendere fl) oF x+2)—VQ4A)+A43)+A—2)=0 6) > shaving infinite numberof soution with = y= se.) rayne 3e end 34 2— (+2) -Q+2) +3) 1 2-1 2 ca. 2) sss tush (5 2 2 a0-2 4c 5) 2 i 2 Now put ), = 5in (i), we get "Now, put the value of Cin (1) TxTy+82430 Now perpendicular distance from (0, 0,0) tothe place 1 3 vaining Land, Se = containing hs da” 38 options ee a PA ey PAPER -1 Es 6 ‘This section contains 7 questions. Each question has 4 options (A),(B), (C)and (D), ONE or MORE THAN ONE ofthese four (ate) correct vy 112x—y+ 1 =Oisa tangent tothe hyperbola ~>—TE =1, then which ofthe following cannot besides ofa right angled triangle? (A) a4t ®) 442 © 281 ©) 24,41 Ifa chord, which is nota tangent, of the parabola hhas the equation 2x +y= p, and midpoint (h, k), then which ofthe following is(are) possible value(s) ofp, h and k? (A) p=-2.h=2,k=4 @) pant. h=L k= © p=2,h=3,k=—4 ©) p=s. 3 Let [x be the greatest integer less than or equals tox. Then, at which of the following point(s) the function x) =x eos(n(x +[x})i discontinuous? (A) x=-1 © x1 o Let £: R=» (0,1) be a continuous function. Then, which of the following function(s) has(have) the value zero at some point in the interval (0, 1)? (A) P=f9) 8 “f(tcostat © eff Kesinvat ©) Hx+J2 AUYsintat Which of the following is(are) not the square of a 3 * 3 ‘matrix with ral entries? Loo 100 w fore @ lore oo1 90-1 1 10 0 © |° w |e to ° 0 0 + Leta, b,x and y be real numbers such that a ~ b= 1 and y* 0. If the complex number z = x + iy satisfies im( 2?) = hn which othe tong iar posible value(s) ofx? @) -yiey? ® a-¥ © ny © Ley Let X and Y be two events such that P(X) al © PKAY=5 =2 @ PRLY)=F Boca ‘This seetion contains 5 questions. The answer to each question isa SINGLE DIGIT INTEGER ranging fom 0109, both inclusive. & Forhow many values ofp, the ciclex? +y?+2x-+4y-p=0 and the coordinate axes have exactly three common points? Let: R> Rea differentiable function such that f0)=0, ((§)->mer Mex) = [[f"(@)eosee t— cot teosect Mt)}at for 2] then lim g62)= 2 IEE Advanced 217 Solved Paper 10, For a seal number a, if the system 1, Woof dar med ing tiles, B,C, F, GH, 1, J. Let x be the number of such words where no La@lx) pr letter is repeated; and let y be the number of such words peat) tt rexel sepa a oe ter i, ye of linear equations, has ee ie iret Then, 2 infinitely many solutions, then I +a 12, The sides of a right angled triangle are in arithmetic progression, Ifthe triangle has area 24, then what is the length ofits smallest side? Exe ‘This section contains 6 questions of matching type. This section contains two tables each having 3 columns and 4 rows. Based on cach table, there ae three questions. Fach question has four options (A), (B), (C) and (D) only ONE OF these four option is correct. (Qs. 13-15) : By appropriately matching the information given in the three columns of the following table Column 1, 2, and 3 contain econies, equations of tangents tothe conics and points of contact, respectively. : @ wey! ; 2) Gi) yom fm? ® (ee a) -a?m -1 09 yom Fa I » (Gas) 13, For a=, ifa angen is drawn toa sitable coni (Column 1) a the point of contact 1,1), then which ofthe flowing option isthe only correct combination fr cbaining is equation? A OO) 8) GQ © MH ©) Mae) 14, Ifatangent to suitable conic (column 1) is found to be y= x + 8 ands point of contact is (8, 16), then which ofthe following ‘options isthe only correct combination? A OWD@ ) MER) © Moe ©) MNGi 15. The tangent toa suitable conic(Cotun tat (55 Ja found to be /3x+2y = 4, then which of the following options is the only correct combination’? (A) avyGin(S) ®) OMGS) © aMg~ © MMR (Gs. 1618) : By appropriately matching the information given in te three columns ofthe following table Letf(x) =x +g, xx log, x, © (0,2) Column 1 contains information about zeros of fx), "(x) and f(x) Column 2 contains information about the limiting behaviour of f(x), fx) and £"(x) at infinity Column 3 contains information about ineeasing/deereasing nature of f(x) and f(x) GP_3480 MATHEMATICS 3 Column Column? Column’ © f)=0 forsomex €(1,e2) @ limf)=0 () _fisinereasing in 0, 1) ()f"(%)=0 for somex « (1,¢) fo _ limf(@x)=-0 (Q) _ fisincreasing in (¢, ¢*) (I F'E)=OFersomex (0,1) —— iy imMECD= =e ®)_fisinoreasingin (0,1) ©) s"@)=Ofersomexe (ey Gv) jim f")=0 (S)_ fis decreasing in ¢, 2) 16. n. Which of the following options is the only correct 18. combination? & OO ®B) GQ © aNainRy ©) (MS) Which of the following options is the only correct, combination? OR ) Uy Giin(s) © aneme oO MOS ‘Which of the following options is the only incorrect, combination? & MGine) ®) MQ © Mow JEE Advanced 2017 Solved Paper PAPER -2 Soros ‘This section contains 7 questions. Each question has 4 options (8),(B), (©) and (D). ONLY ONE of these four options is correct, 1. The equation ofthe plane passing through the point (1, 1,1) and perpendicular tothe planes 2x + y~2z=S and 3x—6y— 22=1,is (A) 14x+2y~152=1 (B) 14x-2y+ 152=27 (OQ Mx+2y+152=31 (D) 14x +2y+152=3 2. LetObethe origin and let PQR bean arbitrary triangle. The point S is such that 08.00 +OR.0S = OR OF + 00.08 = OG.OR + OF.0S ‘Then the triangle POR has Sas its (A) Centroid (B) Circumeentre © Incentre (D) Orthocenter 3. If y = y(x) satisfies the differential equation Sal Wor )ay = (Yerrve] x>Oand y(O)™ 7 then y (256)= 3 @ 9 © 6 ©) 80 4. If f:R > Ris a twice differentiable function such that 1 A(1)= 1, then £(x)>0 forall © Rand f (3) 2 ww rayso ® o1 ‘5. Howmany3 3 matrices M with entries from {0, 1,2) are there, for which the sum of the diagonal entries of M? Mis 3 (126 @) 198 © 18 @) 135 6 LetS={1,2,3,..,9}-Fork=1,2,..,5,letN,bethe number of subsets ofS, each containing five elements out of which exactly kare odd. Then Ny + Ny +Ny +N, +Ng= (210 (B) 252 © 125 ) 126 7. Three randomly chosen non-negative integers x, y and 2 are found to satisfy the equation x + y+ z= 10. Then the probability that is even, is 36 6 w= FF 1 ©3 Peres eeee aoe eee (A), (B), (C) and (D). ONE or MORE THAN ONE of these four pe sin) BA g(xy= JEN sin (Ot, then oe 3m © e(3)-2 e(-3)--2 9 Let a and B be non-zero real numbers such that 2{c0sB — cosa) + ose. cas = 1. Then which of the following, is/aretruc? w =(3-ia() oy n(3m() © =(2)-Haf) o n(2)-m() 10, Iff:R—>Risa differentiable fanetion suc that x)> 2%) forall x eR, and f{0)= I, then (A) fix)isinereasingin (02) (B)_fixisdecreasing in 0,2) © fein.) ©) F) Log, 99 a 50 are eee eects ne oneal eee there are2 questions. Each question has four options (A), (B),(C) and (D) ONLY ONE ofthese four options is crrect. PARAGRAPH I LetO be the origin, and OX, O¥, OZ. be three unit vectors in the @B) 1<10g,99 ot directions of the sides QR, RB, PG respectively, of a wiangle POR. 5 15. |OXxO¥. (A) sin +0) ®) sin2R © sin+R) () sin(Q+R) 16, If the triangle PQR varies, then the minimum value of c0s(P + Q) + 00s (Q-+R) +005 (R+ P)is 5 ™ -> 3 5 ® 5 2 5 © OF PARAGRAPH-2 Let p,q be integers and let a, B be the roots of the equation, x? — hherea# B. Forn=0, 1,2,...,leta,=pa"-+q 8" FACT : Ifa and b are rational numbers and a+b/3=0 then 2 ® ay, + a0 ©) ay, +239 18, Iaj=28, then p +24 = 21 ou oT © 2 6 JEE Advanced 2017 Solved Paper LUTIONS Paper - 1 + Lisatangent tohyperbola 4, 1,8,4,2;28,8,1 ce Bag Ban ras ‘cannot be the sides of a right triangle. If(h, k)is the mid point of chord of parabota y= 16x, eee 2 © 0) But given, the equation of chord is dxty= Q) (1)and Q) areidentical ines sh-K sok Which are satisfied by option (C). 3 (GD) Let x= n be any integer not equal to zero, Then fim_xoostn(x+ x) =ncox(x(n +n) =ncos(Qn—e==n lim, xeos(u(x++ xD) =ncos(n(n+{n})) = ncos(r(n+n)) = nos ne=n LHL # RHL. = limit does nt exist at any non zero integer n. fis discontinuous at x Atx=0, LHL=RHL=0=f(0) fis continuous at: 11,2 4. (A,B) Letus check the given options one by one (A) Letgtx)=2? fx) = a(0)=~f0)<0 Alsog(1)= 1 —f)>0 22 =x) =0 forsome x (0,1) + fi) €0,1) B) Leathiay=x-J2"F costae hO=—[2 F(peostdt<0 and b()=1 [2 "cost at>0 hia) atsomex € (0,1) J? “fancost at =0 © ef FC) sint at + xe@) > &e(.e) and 00¥xe0,1) § (B,D) Inoptions (A) and (C) [A= 1 and in option (B) and (D)[A"|=—1 ‘We know [A?| =A? and|AP#-1 => matrices given inoptions B&D cannot be the squares of any 3 « 3 matrix with real entries. 6 (AB) a ml alxriy)rb (x+t)-iy > Im Ceeiy “(er (ax + b)y+ay(x+1) 2 (x+IP+y? ~axy~by tary tay > (x+1P +y* > abe(tIty GP_3480 MATHEMATICS 7 = leetirty ; — tim y 290 cosx 10, (1) Forinfinite many solutions = =3-£()=3-1=2 = xs 2| : =0 = (l-d¥=0 = a=tl For a= I, the system will have no solution and for PRAY) ‘@-=-1, all three equations reduce to x= y+2= 1 PO)= POXTY) giving infinite many dependent solutions Irata=1-1+1=1 PRAY) 1.) x20! and y= "Cx e os0x91 POM)= Fay 21 Pew) 50x91 9x” oF i 12. (6) Let the sides be a—d, a, a + d where d is positive. Using Pythagoras theorem, 2_7 (at dP =(a-dP +a PROM SS 15-15 = Aan B are the correct options. 8 (2) Centre(-1,-2) Geometrically, circle wil have exaely 3 common 4d Sides are 34, 44, Sd sant = $aaado24 points with axes inthe cases on ie eee p=0 te “Sides are 6, 8 10. i) Touching x-axis and intersecting y-axis at (wo ; points ic, f > Cand g?=C. ae ie 4>—pandl 13, (B) Fora= J and point of contact (-1, 1). = p>-dand Equation of circle is satisfied = pe xty=2 (i) Touching y-axis and intersecting x-axis at two then eqn. oftangent is pointsie. = cand g!>C atye2 > m 5 4=-pand1>-p ‘and point of contact = petandp>—1 ‘whieh isnot possible. {ma _a_\ _(—V vay ‘Only two values ofp are possible ear Teen! (Gi), (Q)is the correct combination. 14. © Tangenty=x+8 => m=1 point 8,16) “both the coordinates as well as m, are positive, 9) Givengo)~0, (5) =3, 10)=1 260)= J2[F(tJeoseet cot coseet (tJ ade sa the only possibilty ofpoint is (S 2) =@,16) 2%)" sim J2 l(t) cose t) dt oars ae Als it sates the equation of curve - (2) cosee™ —Flx)eoeex y= 4ax forthe point (8, 16) And cquation oftingent my =m°x-+aissaisiedby mda’ =3-fpecoeex (ah), Piste cortet combination, 1 18 (D) Pintofcomac (V4) andtangent f5x-+29 = GP_3480 8 JEE Advanced 2017 Solved Paper fx € (1,0, £"(x) <0 = fisdcresing on (1,6) 8 £(x) 20 forsome x ¢ (1,6) 2 dV)istalse. Both the coordinates are positive and m is near negative the possibilities for points. are ‘Also Lien 9)= Hag +(1—x) bog: (i) is false and (ii) is true. ( om a) Ea) een (km rn) (iiyistrue (Ja 2a) r (ivyistrue. For point Q | FJ = (64) fs nctesing (01) aety desl Ifx €@e)then Wegeta= J7 anda= 2 ‘ Pey= 5 —lowx<0 whic is not posible rotate > fisdecreasingin <3 7 af ays 2 | (6 :) (Qiistrve, ForpointR| E34" Vag?aa) “O22 Farx €(0,),1"@)<0 wa Va = Fisdeereasingin(0, 1) 2, 2 Risfae = oi Joel Forxe(6),f"(9)<0 prea ites 2 = Pdecressing in e,e) 2. (S)istr 16, (@) — Theoaly corect combination is), (i), (Q) Aso fora? = 4 equation of ellipse 17, (B) _Theonly correct combination iH, (i), (8) 18 (©) Theony incorrect combination iH, (),(R) Paper - 2 (For questions 16-18): We observe the following in the given + (€) The required equation of plane is given by xtaly I, (iv), Ris the correct combination sats forthe point (3, table ae fis) =xonsnloe, (0) bry ; . b 6 2 FP 3 6-DEH-G-DQ+e-DEIH=0 fU > Oand f(e"): => 14x 14+2y-2+ 152-15=0 fh for some x € (1, ¢") => 14x+2y+152=31 ise 2 ©) OF OG+OR-O8 = OR0F--00-08, Pd)=1>omnar@= 2 -1<0 = (00 - OR) OF (00-08 -08=0 Po) 0fersomex €(1,9 56-7058 -08) <0 iste = (0Q-0R} (oF- 33} 1 > RO-H=0 Ix€ (0,0, + >Oandlogex<0 erate ® Also OR OP +09: OS = OG OR + OF OS P()= + —togx>0 = fisinceasing on 0,1) ee £920 forsomex (0,1) = SRG OR) oP-Fa-0 (ays false = (8-7) (OR 28) -0 MATHEMATICS w © @ => @SR-0 = QPLSR @ From (I) and (2) should be the orthoventre of APQR, Given DE can bewritten as. a Jo- I SeaBeatink puting Yar vbode ~t We get 244 V9e VK 204 Vx 2x Jav=fa = yxtte y= enn +€ Or V7 > y= yar Vorve y256)=3 £()>0,VxER (00 £" isan increasing function on R. By Lagrange’s Mean Value theocem, 2 reorttewmad(!.) = rat a) a) a LatM=|a4 a5 96 | wherea,€ {0, 1,2} a ay a ay ay]; ay ay a as ag|lag as 94 3 a ay|lay ag ay Sum ofthe diagonal entresin M7M=5 > (a) tay? tay) + ay? + ast tag) Hay tag tay)=5 Itis possible when Case f:Saisare | and 4's are zero Which ean be done in 9x8x7x6 aed ‘Then M™M, Ca way 126 S20) vy u = un $4 Vu > Case Il: 1 ais 1 and 1a;is2 and rest. ‘Ta!sare zero Itean be done in °C, «*C, =9 « 8=72 ways Tata no, of ways= 126+ 72 198 Nyt Np +N +Ny +Ng= 126 ‘Total number of non negative solutions of x + y +z = 1are="C, (using"™'C,.) Ifz is even then there can be following cases: 2=0 = No.ofways ofsolvingx+ y=10 = "'C, 2 = No.of ways of solving x +: 45 No.of waysof solving x +: 6 = No. of ways of solving x+: 8 = No. of ways of solving x+y = 10 = No.of waysofsolvingx+y=0 = 1 Total ways when ziseven=11+9+74+5+3+1=36 a00)= fot sn (Q ae in sin 2x) - 2.008 2x —sin"'sin x) - cos x 23) sens: 200e-sint (in) co in“\(sin(-1))- 2eos(—n)— w= (ol) E3) - "None of the options are matching here Itweeonsider an a/2= and an 2 = then 2(cos B — cos a) + cos a cos B= 1 2)-y2 2 fg eg). te ny tee] ely) = Y+x\I1-y)--x) +) (+x) +y)-0-x)(1-y) 462-2) =208 +) x=3y" xe iy 2 (AO) F)-2F()>0 a H(8) _ 45-25 Be 2x) 0 4 [e*t(x)}>0 10 R. => ©" ffx) isan increasing function farx>0 f(x)> £00) = eMYE1 => fiy>e*in(0,20) Also f()>2e">0 fisan increasing function in (0,2) (0) t= ty HOME) pl = int “heh he i) -ino() im, x)= lim rat = eh iM (1m) (2) h -anio-aield '=-2» (Some value oscillating between =I and 1) does not exist. leos2x cos2x 08x c08x sin2x ~sinx (BO) fi) = a= fi-07i= Operating C, > C,-C, 1 = 5ca J025 = 7 alsoa<1 sin2x sins tlk +1 osx 14. (B,D) I= zi wey = 1X)=2¢0s3xc0sx Latx-k=t > dade => fx) = cos 4x + 005 2x, ren fgg =2 atx=0 5 Siete Tarr 1 (x)=—4 sin 4x—2 sin 2x, . 2sin 2x (4.008 2x +1] . fl et Eee £@)=0 = sin2x=0 oF cos2x ‘a = b Dk] x i $00. which struc for some x €(-x,) £ (x)= Oat more than three point in (x, x) =» Ban(-zy) GP_3480 MATHEMATICS u 16, (B) _cos(P+Q)+ cos (Q+R)+cos(R+P) :05 (180) +605 (180 ~P) + cos(180—Q) [cos P+ cos Q + e058 R] In any APQR, cos P+ 60s Q+ cos R< => ~(e0sP + 608 Q+ 60s R)>— Required minimum value =~ 17, B) = a, arerocts ofx?—x~ @—a-1=0,6°-B-1 5 a@=a+tandpi ‘Also a, =pa+qB" > % =p+q a) =pas ph ay = pal +B = plat 1) +q(B+1) (a+ aB)+ (p+ 9)= 41a a spa? +qf*=pa(a+1)+qB(B+ 1) (pa? +98") + (pa+aB) Proceadingin the same manner, weget 1D) ay =aytaznay tay tay aya, . = 3a) + 2a 18. (A) OX, OY, OZ are unit vectors inthe directions of| =3(pa+4B)+29+@) sides OR, RP and PG respectively, af (149), 0(1-v8) > PES] 0-0 1, 3 = }lo+a)+5(0-a)v5 =28 > pran4 pt2q= 12 P+Q Q x sin P+Q) ogredo Held On Let k be an integer such that triangle with vertices (k, -3k), (5, k) and (-k, 2) has area 28 sq. units. Then the ‘orthocentre of this triangle is at the point © (23) @ (2-4) ° (3) (3) If, fora positive integer n, the quadratic equation, OCH 1) HOC TOC 2) aan A(R DHT) (KA) = 100 thas two consecutive integral solutions, then nis equal to ou @ 2 6) 9 @ 10 retain | ()) neither injective nor surjective 2) invertible {G) injective but not surjective surjective but not injective ‘The following statement @>9)>1Cp>9) >a) is ()afallacy 2) a tautology ). equivalent to~p—q (4) equivalent to p—>~4 [FSi thet of distinct vales of "bfor which the Following system of linear equations xtyt xtayte axtby+ hhas no solution, then S is () a singleton )_ an ininte set (finite set containing two or more elements The area (in sq, unis) of the region {Gx,9):x20,x+y<3,x?) 1, Hhy=atanset bx +C where C is constant of integration, then the ordered pair (a,b) is equal to w (+49) 1 o (54) Let obea complex number suc that 20+ 1 =z where boi 4 Br 07-1 0?|=36, then kisequalo 2 gl ail Q) 2 z @4 The value of lc, —9C,)+ AIC, -C,)+ IC, MC,)+@IC,— MC) FoF CCyg—MC is @ 2-2" ) 221-211 @) 2-2" @ Mm” cotx—cosx Tim S= 298% couats rok (2a) 1 1 1 1 OF Oy Oe OF IF S(tan?x—cos?x)= 2e052x+9, then the valve cos 4 is 3 1 2 M-3 @ 3 @s If the image of the point P(1, -2, 3) in the plane, 2c+3y-4e 422-0 meadpanliotng © Q, then PQis equal to O65 ® Wa The distance ofthe point (1, 3, ~7) from the plane passing through the point (1, ~1, -i),having normal perpendicular to both the lines @ ® 24, 25, a. 2, GP_3480 1 if 6x) (1g) te deve of en 855) J 9(x), then g(x) equals: : . o 149x3 @ 149x> xv 3x % 1-9x3 @ 1-9x? ‘The radiusofa circle, having minimum area, which touches the curve y= 4~x? and the lines, y= |x| is: @ 421) @) E41) @ 3-1) @ 42-1) A box contains 15 green and 10 yellow balls. £10 balls are randomly drawn, one-by-one, with replacement, then the variance of the number of green balls drawn is 6 RB ” OF 6 ws 25 “The eccentricity of an ellipse whose centreis atthe origin i totes 4, onteogtineti (I) x+2y- 3) is 2 6) ax} @ axtdy=7 iw diferent numbers are aken from the set (0, 1,2, 3, 10), then the probability that thet sum as well as absainte difference are both multiple of 18 7 6 O® as 2 4 os 9S For three events A, B and C, P(Exactly one of Aor B occurs) (Exactly one of B or C occurs) (Exactly one of C or Acccurs) 1 PA the three events occur simultaneously) = ‘Then the probability that at least one of the events ‘occurs, is 3 1 a 16 Q D oz od is a 23 uta=[ 2, 7 fehenadian? +12) iseqato 63 n -8 s © [ssi n ® [a St 68 si at o [i o [sal MATHEMATICS SOLUTIONS 1. (1) Wehave ; Be » = 5k2+ 13k-46=0 or E+ 13k +66=0 Now, 5k2-+13k—46=0 i320 f S \. =k 10 5.2) 2.2) 3 ke Dina? since k isan integer, -.k=2 ‘Also Sk?+ 13k +66=0 op TI 10 Sono ra solution exist, Fororthocentre BH LAC (S = a-28=1 “ AAlsoCH 1. AB (S33) > @ Solving(1) and (2), we get 1 a=2,B=5 etter (2 2 (1) Wehave ¥ (c+r-t+r)=10n a > (2 + xr-+(F=Dx41?—1)=100 at Berne at Sol (143454..+@n-1)}xt (124234... 4(-1) n}=10n Sot te SoDMOeD stent Let cand a.+ 1 be its two solutions (Cz ithas two consequtive integral solutions) Sat(atl=-n =n-1 san a ?-31 Also a (0+) 2 3 Putting value of (1) in (2), we get (es! 3. => (x) changes sign indifferent intervals Notinjective Rangeis = Surjective but not injective 4.) Wehave Dla=ppsa[=ps ap 0a [P03 tlre |e | ot F T titfe | t | ot r T Flrlr | 7 | & T 1 eirlr | | r 1 aa @ It's tautology. First two equations iextytz=l Tohave no solution with x-+ bel b]=1 [1a] +1 [b- are identical by+z=0 Sob= {1} = Itis singleton set Area of shaded region 2 (1+ Vieja +f B—x)ax— 1 3 x 7 2h We have 9(25a2 +b?) +25 (c2—3ac)= => 225a? + 9b? + 25¢? - 75ac: 43x} => (15a)? + bY? + Se)? —75ae—45ab— 15 be= 0 a2 [pe a IsbGa+e) = 45ab + 15be + [(Sa— 3b)? + Bb—Se)? + (Se—15a)?]=0 itis possible when 1Sa~3 =>b,c,a arein AP. 3b-Se=Oand =>(a-1)?=0 GP_3480 JEE MAIN 2017 Solved Paper Possible cases for X are (1) ladies, Oman (2)2ladies, 1 man 3) ady,2 men G@)Oladis, 3men Possible eases for ¥ are (1) Oladies, 3 men (2) lady, 2men (G)2iadies, man (4) ladies, Oman No.of ways =4C,.4C,+ Cg. 3C)E+ Cy ICQ)? +0cyP = 1643244 144+ 12485 x46 Wehave y= 2¥x=3) Atyaxis, x= On dilreniating, we get 2 5x46) (1) (+6) (2-5) & (P+ FE = Latpoint (0,1) Slope ofnormal=—1 ‘Now equation of normal y— Sy-l=-x xty=l (jaws Equation of hyperbola is ~ areas foci is (£2, 0) = Since b? = a @ @ Sbleb1=0 = (b-3)(+4)=0 > Bas & Fort? 4 (Not possible) MATHEMATICS airs 2 1B. @ @ 4 wix_ sly Equation of tangent is ~T*—*>* sune=t Clearly (22,33) satisfies it. 8 flx)=ax? +bx +e a) d A) a= SFT mbt thy Q) A ’ Subuact 2 from (1) es Gy23+44+5+...uptonterms asin (n? +5n) ; n(a.+p)= 48 2 _, tanattanp_ 1) “HNGTP)= Ap (o2+5n) tna Le =D > Fema 2 aarp)! [From] j[pesnane, Satan) n(n +1Xn+8) : 6 2) 6 oo lox nas s—_ Sw" 30 —Tanp 2 4 . +r+10=20 i 2) +k >u+19=20 BEY: 20-2r laxbe v2? 4274? = 0 ‘We have | (xb) xi: by) é| sin 30° Aa (2) . AwAtcan Ponts boat (20 lero Aq 10r-7 ForAtobemaximam =3=3)é)4 2 ; ' le|=2 Now |¢=a On squaring, we get aera Dea 449-2 = = ag72 tl Apt since AP=20B = AB + ” AP 2 16. @ Ie 5 5 Using ff(x)dx = [fla +b—x) dx ‘Adding () and (ii) ccosec?xdx I= (eotx)yidt =-[os fs Wehave 2+ sinx) $2 +(+ Deosx =0 2 devinntyy-0 On integrating, weget Q+sinx) (y+ D=C Atx =0,y= 1 wehave Qrsino(l + H=C 4 Desinx = fica +8 ade fin! ase? xd Leltanx=t => see? x dx = dt @ 18 @) 1». a) 20. @ a. a JEE MAIN 2017 Solved Paper >0= => wis complex cube root of unity Applying Ry > Ry + Ry+R 3 0 0 1 -o=1 oF? 1 @ © 3 (-1-@-@)=—3 (1 + 20)=—32, Webave lc, #21c, +21C\9) = (9C, #99. !9Cyg) = HOC + sey) ¥AlC, + CygI-2-1) 194 Cy BC yg=2"°— 1) = 4ot-2-2"-p =(20—1)- 1-1 20 _ p10 Tim £0020=Si02) «jm SOERC1=sinx) soap ee) tant(1-cost) oo ae tant I-cost z 16 We have Stan? x~S eos? x=2(2 cos? x-1)+9 > Stan? x-S cos? x=4 e082 x-2+9 Stan? x=9 0s? x+7 GP_3480 MATHEMATICS main 2. 2 m4. 8 ® @ = 5 (see x— 1)=9 008? x+7 Let cos? akan 9+ 12t-: 398+ 15t-31-5=0 SGt-)Gt+5)=| stetate3 ater g 082x= 2.005? x— i Equation ofline PQis 25, (None) wot ys? 7 Let F be (2+1,44-2,5243) PQ,-2,3) NF No | ee “2 (+ 1)+3 (4h=2)—4(52+3)+22=0 2424 122,~6-20A~12+22=0 3 -6.+6-0>4=1 Fis(,2,8) PQ=2PF=2 Ji? 44? 45? = 282 Let the plane be > aG—-D+bytHee@+)=0 Normal vector > aE <. Equation of rele becomes 1-2 3 [esietjeak eiy_pre® » ta O-KPe > o Soplancis(x—1)+7(y+1)+3(@+1)=0 Ittouches y=4—x2 as well S54 Ty+3z+ «Solving the two equations Distance of point 1, 3,7) from the plane is eB oa2ie Zits ean = z T5r4909 re | ( 6okk fi = seyakene E+ ao taro) 5) tex (04) i gveenoasD=0 2.x?) 2oer] =2tan-! Gx) 6. m. 28, ke Ee ‘Which isnot matching with any of the option given here. 2) Wecan apply binomial probability distribution Wehaven=10 @ Wehmeb!=2-e)=2(1 js ~ Equation of ellipse is 2 a Now differentiating, we get x 3 Slope ofnormal=2 Equation of normal at (3) is @ denotes sample space Let E bethe given event Ex {(0,4).(0,8), (2,6) (2, 10),(4,8),(6,10)} 2», 30. @ @ GP_3480 JEE MAIN 2017 Solved Paper P (exactly ne oF Aor Bet) 1 =P(A)+PB)-27(A B= 4 o (Exact one ofB or C occurs) =P@)+PO-2PHAC)= 1 2 P(Esaetly one of Co Aces) =RC)=R(A)-2°(CHA ® a ‘Adding (1), (2) and (),we get 2EP(A)—2EP(ANB) I 4 ma-manne! 1 Now, PAAMBNC)= ig P(AUBUC) =3P(A)-2P(ANB)+P(ANBC) 3,17 ais 9 eXsli2 13 ».[ 48 27 S30" | 36 39 4 36 Also 12A=| “4g 1 (5 SMe He 2 ~ BAP 2A, 36 39)*[-48 12 J[-86 st st 68 adj GA+124)=| 54 9 CHAPTER Trigonometric Functions & Equations I Section.a 4’ Rr Suppose sin?xsin3x= 5° Gq cosmeisan identity in x, ee where Cy, C,, .C, ate constants, and C, #0. then the value of is (1981 -2 Marks) 2n 2 The solution set of the system of equations x + y= cosx-+e0sy=3, where x and y are real, is (1987-2 Marks) 3. These ofall x in the interval [0, x ] for which 2 sin? x3 sinx +12 0,is (1987-2 Marks) 4. The sides ofa triangle inscribed in a given circle subtend angles a, 8 and y at the centre. The minimum value ofthe arithmetic mean ofcox( a + 2) as(-+) and con(+ ) is ou to (1987-2 Marks) The value of 5 in 78 gin 2 gin sin a ae to (1991 - 2 Marks) 6 If K ‘numerical value of K is Be is equal sin(x/18)sin(Sx/18)sin(7x/18), then the (1993- 2 Marks) 7. If 4>0,B>0 and 4+ B= n/3, thenthe maximum value (1993-2 Marks) 8 Genoral value of 0 satisfying the equation of tan 4 tan B is, tan? 0 +30¢20= 1 is, (1996-1 Marky 9. Thereal roots ofthe equation cos? x + sin x1 inthe interval (Ca,)are.....,and (1997-2 Marks) 8 Ei If tan 4 = (1-008 B)/sin B then tan 24 = tan B. (1983-1 Mark) There exists a value of @ between 0 and 2x that satisfies the equation sin 9-2sin?0—1=0 (4984-1 Mark) [el] MCQs with One Correct Answer Ittand = 4, then sindis 1979 3 4 4 a) — 4 butnot by - (@ ~F butnot 5 © 4 4 ©) = butnot— 4 (&)_ None of these. (© butnot— 5 @ "l Ifa+B+y=2n, then 0979 B (tan +tan Han B tan ® tan + tan tan ( tan$ tan 5 +tan5 tant +tan 2 amp tiny 2 © stan 8 +t0n t 8 (© tan +tan 5 +tan tan tan (None of these. Given A=sin? 0+-cos*@ then fora real values of 8 3 @ 1s4<2 () Fs4sl 98 3B © igs4s! 1e equation 20s? sin? x. The equati : (@)_noreal solution (6) one real solution (©) morethan one solution (4) none of these “The general solution ofthe trigonometric equation sin x*eos (1980) x= Lisgiven by (@) x= 2nm 570, H1,42 () x=2nn+n/2;n=0,41,42. (1981-2 Marks) © xemrey" m0, 4 (@ none of these 10. 2B. 4. oe __ topic-wise Solved Papers - MATHEMATICS ‘The value of the expression 3 cosec 20° ~sec 20° is equal to (1988 -2 Marks) @ 2 (b) 2sin 20"/sin 40° © 4 (@ 4sin 20%/sin 40° ‘The general solution of sin x~3 sin 2x + sin 3x= c0sx~3 cos 2x+ c0s 3x is (1989-2 Marks) @ me © © cree u F (© 2nn+ 005" ‘Theequation (esp 1)32 + (es py+sinp=0 In the variable x, has real roots. Then pcan take any value in the interval (1990-2 Marks) © @29) © Cn © (-23) @ Oa Number of solutions of the equation (1993 - 1 Mark) tan x + seex = 2cos x lying in the interval [0, 2] is @) 0 (b) 1 () 2 @3 Let 0 3(sinx~cos2)4 +6 (sinx-+e082)? + 4 (sin x +c08° x) = (19988) ®n ®2 @B @ 4 ‘The general values of 6 satisfying the equation 2sin?0 —3sin0-2=0is, (19988) meen" a/6 () mme(-I)"x/2 (©) nee(-1y'sn/6 (mee (-1)"7R/6 15, 1". 18, 1. 20, 21 2. ston oo istrucifandonlyif (1996-1 Mark) wey @ xtyz0 () x=y,.x70 © xy (@) 340,920 Inatwiangle POR, ZR=n/2. Man 2) and tan (QI2) ate the rots ofthe equation ax? +bx-+e=0 (a:#0) then (1999-2 Marks) @ ato-c © b+ema © atend (@ b=e Letsi0)= sind(sin0 +sin30).Then/(0)is (20008) (@) 20 only when 920 (©) 0 forall real 9 (©) <0 forall real 9 (@ <0 onlywhen 0<0 lsinx cosx cos] The number of distinet real roots of 008x sinx cosa} 001s) @o 2 © 1 @ 3 The maximum value of (cos a,)(c0s a,)...(c0sc,), under the restrictions =Oin the interval —7 xs is Oey, a,--.0,$ % and cota,)(cot4,).. (2) is en) @ 12 & 1 ©) Im @ I Ifa+B=s/2andB-+7=a, then tan aequals (a) 2(tanB +tany) (b) tanB+tany (©) tanB+2tany (@) 2anp-+tany Themumber of integral values of fr which the equation 7 cosx +5 sin. 2k + | has a solution is (2002S) @4 8 © @ 2 (20018) ore LaeaberuetOs tte Om = w (34) ‘x @ (Bx .cos(a~ )= 1 and cos(a+ B)= Ie where a, B « [-r, 7]. Pairs of a, f which satisfy both the equations is/are (20088) @o wml ©) 2 @ 4 The values of € (0, 2x) for which 2 sin?0~ 5 sin0+2>0, (2006- 3M, -1) GP_3480 28. Let oe(0.2) and f, = (tandy™®, ¢, = (tans, 15> (cot0)"® and f, = (c0t8)°, then (2006 - 3M, 1) @ 4>Q>n>y ©) >nryry © K>A>nee @ g>A>n>4 26, ‘The numberof solutions ofthe pai of equations 2sin8 ~cos26=0 2cos* — 3sind=0 in the interval [0, 27] is (2007-3 Marks) (a) zero (b) one (©) two (@) four 27, For x ¢(0,) the equation sins + 2sin 2x~ sin 3x=3 has (EE Adv. 2014) (2) infinitely many solutions (©) three solutions (©) one solution (@) no solution 28. ase {x (-a,n):x 40, } ‘The sum ofall distinct 2 solutions of the equation V3 sec x + cosee x + 2(tan x — cotx)=OinthesetSisequalto EE Ad. 2016) I 2e ® -> © -> Sn © o oF 2 1 29, The value of > is equal 0 (JEE Adv. 2016) @ 3-5 @) 2(3-3) © A3- @ 2(2-V3) B» J MCQs with One or More than One Correct ® an $n In 140082 {14-0828 \ 1+ 0052" {1+0087=) is equal ' ( il 8 8 7) plauel © (1984-3 Marks) 1 x @ 5 () ese 1 Li © § @ oF 2. Theexpression 3[snt(25-a) estore] 1[snt{Z+0}ssin6Sx-) ] esmato (1986-2 Marks) 10. (@ sinda +e0s6a, (©) none of these ‘The number ofall possible triplets (a,, 83) such that ay + a, cos(2x)+a3sin“(x)=O forall xis | "(1987-2 Marks) (@) zero (0) one (©) three (@ infinite (@) none ‘The values of lying between 0=0 and 07/2 and satisfying the equation (1988 -2 Marks) T+sin?@ cos?@ sind sin?@—1+0087@ — dsin4a | =0 are sin?@—cos?@ 1+ 4sin 40 (®) 7x24 (b) 5x24 (0) I1n24 (@) nA Let 2sin2x+3sinx—2>0 andx?-x~2<0 (ris measured in radians), Then x lies in the interval (1994) @ (@ %) OO) (1 2) © oa (22) The minimum value ofthe expression sina+sin+siny., where a, B, 7 arereal numbers satisfying a+ B+ 7=7 is (@)_ positive () zero (1995) (©) negative @ 3 ‘The number of values ofx inthe interval [0, Sx] satisfying, the equation 3 sin? x—7sinx+2=0is (1998-2 Marks) @ 0 Os © 6 @ 0 ‘Which ofthe following number(s) is/are rational? (1998 -2 Marks) () 00s 15° (©) sin 15° cos 75° (1999-3 Marks) (a) sin 15° (©) sin 15° cos 15° Feeapeitve integer at 1) -(~ 3} Th + soc0) (1486020) (+5604)... 3602"0), 0) @ a7 nL n 1B. 4 oe __ topic-wise Solved Papers - MATHEMATICS For 0.<0-<, the solution (s) of o<0<5, 6) istare) (2009) x 2 x se ®F oF oF OF Let 0, @ € [0, 2n] be such that 2 cos@ (1 ~ sin @) = sin? 2, 2 an +cot® cosy —1,tan(2n-0)>0and (: 8 cot$ .9-1,tan(2-8)>0. x x ae @ o<00,im(@)>0 The inequality [2~4] <|=~2] represents the region given (1982 - 2 Marks) () Ree@<0 (@ none of these If z=x+iy and @=(1-i2)(2~i), then |oj=1 implies ‘that, inthe complex plane, (2) zlieson the imaginary axis (©) =lieson the real axis (©). zlieson the unit circle (@) None of these (4983-1 Mark) oy % 10, Rn, 1B. 4. 15, 16. 1”. oe __ topic-wise Solved Papers - MATHEMATICS The points ,, zy, Zn the complex plane are the vertices ofa parallelogram taken in order ifand only if (1983 - 1 Mark) @) a 447A +h © 5 + © 342 (©) None ofthese Ifa, candu,v, ware complex numbers representing the vertices of two triangles such that = (11a rhand w= (IP ry, where risacomplex ‘number, then the two triangles (1985 -2 Marks) (@ havethesamearea — (b) aresimilar (©) are congruent (0) none of these Io (¢ Disacuberootofunityand (1+) =4+ Be then Aand B are respectively (19955) ®t OL © 10 @ -11 and « be two non zero complex numbers such that © [and Argz+ Argo =n, then zequals (1995S) O-0 O@ @ -3 Let and o be two complex numbers such that |=| < 1, | |< Land|z+io |=|2-1@|=2 then zequals (1995S) (@) Lore) for-s_ (©) Lor=1 @) For=1 For positive integers m,n the value ofthe expression (ea +4 PY" +4)? + (407)? where i= YT isa real number ifand only if (1996-1 Marks) @ n=m+l 0) m=n-l © mon © n,70,n,>0 won cranes a( 148) is equal to @) 1-3) -14N3 ©) WS ‘fare(2)<0, then arg ¢-2)-arate)= (1999 - 2 Marks) @ -N3 20008) @r & © > oF arecomplex numbers such that (20005) hey then fa +23 42 (@) equal to1 (©) greater than3 Let z, and z, be n roots of unity which subtend a right (b) tess than 1 (@ equalto3 angle at the origin. Then m must be ofthe form (2001S) (@) 4k+1 () 42 ©) ARH3—@) The complex numbers z,, z, and =, satisfying are the vertices ofa triangle which is 22783 (@) ofarea zero (b)_right-angled isosceles (©) equilateral (@) obtuse-angled isosceles 001s) 18, 19. 20. 21 2. 4 For all complex numbers z,, z, satisfying +12 and |zy3-4/|=5,theminimumvalieafiz,lis (20025) @ 0 @2 o7 @n eens @o © ~ (20038) len? lentP Ifo (¢ 1) bea cube root of unity and (I + 0?)"= (1+ 0", then the least positive value ofn is (20048) @2 3 © 5 @ 6 The locus of which lies in shaded region (excluding the ‘boundaries) is best represented by (20058) (a) =:[e4 11> 2andarg (+1) <1 (@) 5-112 andlarg(-D] lie on = (@)_aline not passing through the origin (2007-3 marks) © vt (©) thexaxis (@ they.axis Apatite P starts from the point =1-+2f, where i= JI Itmoves horizontally away from origin by 5 units an then ‘ertially away from origin by 3 units to reach a point =, From; the particle moves /2 units inthe direction ofthe vector /+] and then it moves through an angle I in anticlockwise direction ona circle with centre at origin, to reach a point z, The point zi given by 2008) @ 6+7 () -7+61 © 746 @ 6471 4s £08 8 + isin 6. Then the value of > Ims?"“!) at 0=2%is (2009) 1 1 sane Janae ane Letz=s + pbeacomplexnumber wheexand ringers, ‘Then the arca ofthe retanle whose vertices ae the ons ofthe equation: 223 +22) =350is 2009 @ & (b) 32 ©) 0 @ 9 Let bea complex number such that the imaginary part of= isnon-zeroanda== +2+ | isreal, Then a cannot take the “ value 2012) 1 1 3 @12 %3 OF @| 1 Letcomplox numbersand = ican Seles (%9P +(Y— yp)? =? and (x xp)? +(Y— yp)? = 407 respectively I= %q + i satisfies the equation eal? = +2. then (WEE Adv. 2013) 1 1 ® ® OF 3 al oi 1 £3 MCQs with One or More than One Correct} If that numbers w; =a-+ic and w, =b-+id satisfies — (1985-2 Marks) (©) [mF 1 (@) none of these a+ib and 2 -+id are complex. numbers such ‘and Re(z, 7p )0, then the pair of complex @ [mri (©) Re(mmm)=0 Let =, and z, be complex numbers such that = # =) and 2a -Ithas postive real partand has negative +2 {imaginary part, then maybe (1986-2 Marks) ak (©) real and positive (@)_purelyimaginary (@) 210 (©) realandnegative (6) none of these. If = and zp are two nonzero complex numbers such that Je+2y l1=i|+[=2 hthen Arg =~ Arg =p is equal to (1987 -2 Marks) (@) =n b) ae © 0 (@) 2 On : rwevaicat$ (in22 sox) i (1987-2 Mak) kel 7 (@) -1 (b) 0 © =i @i (©) None If is an imaginary cube root of unity, then (1 + @-«?)" weak aati (a) 128 —(b) =128m (c) 1280? = (d) ~12807 a Thevalueofthesum © (#++#"), wherei= Jj ,equals (1998 -2 Marks) @ i Or O74 @o or 3h ir]4 34 +i then (1998-2 Marks) 0 3 () x=Ly=3 @ x=0,y=0 Let , and 2, be two distinct complex numbers and let fe, foesome real number t with O<¢= 1. ATE (60) denotes the principal argument of «non-zero complex umber then e010 @) |2-2|+|2-2 ©) Are (@ Arge-2)= Arg 10. and P= {w":n =1,2,3,..}- Further Hy = ae foc:me! a tee setofall complex numbers. If 1 PCHj224 POH, and Orepresentsthe origin, then Z2,0z)= (JEE Adv. 2013) PB B em BP oF oF oF oF Leta beRand a+ #0 suppose $={Zec:z=—veR,re0h, whe suypne s-[zces2=cto ected}, whee Fi Ifz=ntiyandz 6, then (x, les on (JEE Adv. 2016) en eee eee ee bro 1 (©) the circle with radius—>— and centre a<0,b#0 (©) thexaxis fora #0,b=0 (@ they-axis fora=0,b#0 © Beato inthe forms iy. Express (1978) T=e0s0+ 2isin0 Mx=a+b,y =ay+ bp and: a + by where y and Pare the a+b (1978) ‘complex cube roots of unity, show that xy FFind the real values of x and y for which the following 30941 _ | 1980) 3-1 uation is satisfied (+9 — 24 equation is satisfied (0 Let the complex number z,, 2, and 2, be the vertices of an equilateral triangle. Let be the circumoentre ofthe triangle ‘Then prove that 2+22+22=322. (1981-4 Marks) Prove that the complex numbers, rand the origin form an equilateral triangle onlyif Ifl,aj,a3 (1-a)t—4) da) Be (1983 - 3 Marks) 14q_ atethe n roots of unity, then show that (1-4,_:)=m(1984-2 Marks) 8 9. 10. 2 13. 18, oe __ topic-wise Solved Papers - MATHEMATICS wB+i 2 Show that the area ofthe triangle on the Argand diagram formed by the complex numbers , igand + izis due (1986-24 Marks) +67.1FZisany complex number Let Z,=10+6rand. such thatthe argument of [2=2)) is ® , then prove that (ZZ) "4 |2-7-911= 3V2 (1990 4 Marks) Ifie)+2*~2+1=0, then show that |z|=1 (1995 - § Marks) If|Z[S1, [WIS show that [Z-WP S((Z|-|W IP Arg Z~ Arg? (1995-5 Marks) Find all non-zero complex numbers Zsatisfying Z =iZ?. (1996-2 Marks) and , be roots ofthe equation =*+pr+y=0, where the coefficients pandg may be complex numbers. Let A and B represent z, and, in the complex plane. If AOB=01 +0 ‘and O4 = OB, where Oisthe origin, prove that p= Aq 00s? (3) (1997-5 Marks) For complex numbers: and w, prove that? w+ ifand onlyit: (1999-10 Marks) Leta complex number a, a+ 1, bea root ofthe equation 2-8 -21+1=0, wherep,qare distinct primes. Show thet cither I+a+a2+... +0? !=Oorlta+a?..ta"!=0, Dut not both together. (2002-5 Marks) Iz, and =, are two complex numbers such tht =| <1< [5] then prove that (2003-2 Marks) Prove that there exists no complex number = such that where |a) <2. (2003 -2 Marks) Find the centre and radius of circle given by kel tiy,a=a, ay, B=B, + 1B,(2004-2 Marks) fone the vertices ofthe square circumseibing the circle fe—1| = 42 is 2+43 1, Find the other vertices of the square. (2005-4 Marks) GP_3480 Complex Numbers F Eo MORE statement(s) in Column-Il. The appropriate bubbles corresponding to the answers 10 these | OOQQO ‘questions have to be darkened as illustrated in the following example (QOOOO Ifthe correct matches are Ap, sand t B- andr; C-p and q; and D>s then the correct darkening of — DI ODOOO bubbles will look lke the given. 1. = Oisacomplex number (1992-2 Marks) Column (A) Rez=0 ®) Ag: 2. Match the statements in Column I with thos in Column 201) {Note : Here takes values inthe complex plane and im. and Rez denote ,espetively, the imaginary part and the real part of] Column 1 Column It 4 (A) The set of points z satisfying (p)_anellipse with eccentricity |r-i1z||= + |= lis contained in or equal t0 (@ thesetofpoints= satisiying Im==0 (B) The set of points z satisfying (©) the set of points z satisfying |Imz|<1 [ed] +24 [= 10 contained in or equal to (©) If|w|=2, then the set of points (8) the set of points z satisfying | Re z|<2 is contained in or equal to then the set of points (9. the set ofpoints = satisfying |=|<3 ©) It} 1 2=w+ © is contained in or equal to DIRECTIONS (Q. 3): Following question has matching lists. The codes for thelist have choices (a), (0) c) and (@) out of which ONLY ONE is correct. watt ust 1 Foreach there exits as such hat p51 1 Te Q. Thereexitsa k ef1,2,...9} such that 2,2 2 False has no solution = inthe set of complex numbers R 1 s. 42 PQRS PQRS @ 1243 2134 @ 1234 @2 143 ee @ __topic-wise Solved Papers - MATHEMATICS fel Comprehension Based Questions PASSAGE-1 Let 4, B, Ce three sets of complex numbers as defined below y -i|=3) A= {ecm B C= fz:Re(l—i)2)= 02} 1. Thenumberofelements inthe set ANB is (2008) @o0 @1 @2 @ © 2. Letzbeanypointin AA BAC ‘Then, + 1 ~i?-+[e~5~i? ies between 2008) (@) 25and29 (©) 30and34 (©) 35and39 (@) ands 3. Lot z be any point AM BNC and let w be any point satisfying jw-2— <3. Then, l|—[w-+3 lies between (@) —6and3 @) Bando 2008) (©) Gand @ Sando PASSAGE2 Let S=5, 05, 05;, where Cif |< 4}, Sy and $3 = {2 €C:Rez>0} 4 Arcaof= (086 At 2013) Wor gy 2g Me 2 oF oF oF oF | cc JEE Main / @IEEE 1, zand ware two nonzero complex numbers such that |=|~|»{ andarge* Arg hen os [202] @ 6 &)-6 Oo @ -o 2. If|z—4|<|z—2|, its solution is given by. (2002) @ (©) Ree)>3 3. Thelocus of the centre ofa circle which touches the circle 2-z,|=aand|z~z,|=bexternally — (c, =, & z, are ‘complex numbers) willbe [2002] (@) anellipse (0) hyperbola (©) acircle (4) none of these 4. Ifzando are two non-zero complex numbers such that 1 and Arg(=)—Arg(o)=% then Zo is equal to {2003} @ 1 @- @i GEE Adv. 2013) o @ 344 2 $4 Integer Value Correct Type Izisanycomplex number satisfying 3-21 < 2, then the ‘minimum value of2=~6+ Sis @oily Let @=e3 anda, b,c, x,y, zbenon-zero complex numbers such that Con) atbrens a+bo+co?=y a+boF+co 2 ‘Then the value of +L is [aPa[bP le Sosa-as| i= JET. The value ofthe expression Flows-ou-2! i is (IEE Adv. 2015) Let Z, and Z be two roots of the equation 2 +a2+b=0 »Zbeing complex. Further , assume that the origin, Z; and Z forman equilateral triangle. Then 12003} 46 ©) a? 2b a? =a then 2003} (a) x=2n-1, where nis any positive integer (b) x=4n_, where m is any positive integer (©) x=2n, where nis any positive integer (@)_x=4n+1,, where n is any positive integer. Let zand wbe complex numbers such tha argzw= x, Then arg zequals se ye >= wt oF OF OF OF GP_3480 aE Trigonometric Functions & Equations I Section-a MRIS ETS WEES x 2 1. If tan A= (1-c0sB)/sin B , then tan 24 = tan B. 1. Suppose sin? xsin3x= }° Cy, cosmeis an identity in x, (1983 - 1 Mark) pes a NY INS 9 Thereexistsa value of © between O and 2 that satisfies the equation sin* 0—2sin? 0—1= 1984-1 Mark where Cy, Cj, Care constants, and C, #0. then the pith at e y value of mis (EEE MCQs with One Correct Answer 2e A 2. The solution set ofthe system of eamtions x+y= "8, 4, tao 4, then sind (1979) 3 4 4 4a4 cos+c08 y=, where xand y are real, is (@) — 5 butnot = tort osx-+008 y= 5, wherex and y are real, @ -5 ; &) -~ Fort Caer © Founo-4 (@ None of these 3. These ofall xin the interval [0, x] forwhich 2sin?x—3 yeas on tn am) sinx+1 2 O,is, (1987-2 Marks) as An 4. The sides ofa triangle inscribed in a given circle subtend (@) tan stant +tand tan tanf tant angles a, and at the centre, The minimum value BoB £ tant tan® tan tant / ) tan$ tan F+tanF tan 2 tant tan = 1 of the arithmetic mean of eos{ a + = 2 © on! stan 2 tant tan 2 tan? (©) tan +tan $ +tan ¥ =—tan$ tan tan (@ None of thes. ooa( © 2) is ual co (1987-2Marks) 3, Given 4 sin? 0+ cos*0 then forall el values of 5. Thevalueof @) 1<4s2 © isasi (1980) in 38 sn sn ™ gin sin! 1 @ Bsasi @ 3sas8 sin sin Sin Sin in sin! in 3 yu i eee © (1991-2 Marks) 4, The squaton 2e0s? sin? x=? 4%, Oc bas 6 Mf K=sin(n/18)si(Sn/18)sin(72/18), then the (@)-noreal solution” (6) onereal solution sumerical value ofk is (1993- 2Mtanty) (©) morethanonesoltion (@) none ofthese (7980) — The general soon ofthetigonometi equation sin.ros 1. 4>0,8>0and A+B=2/3, thenthemaximumvalie "=f isgivenby (1981-2 Marks) ofan Aan Bis (1993- 23Marks)——@) x= 2am 00,41 8 General value of satisfying the equation r=2men/2n tan? 045220 1s (1996-1 Mark) 9. Therealrontsoftheequationcos?x+sin¢r-lintheinival = (@) = AEH" (Cy mpare, and, (1997 - 2 Marks) (d)_none of these EBD_7202 10, 2 1B. 4. trop ic-vise Solved Papers - MATHEMATICS. ‘The value of the expression [3 cosec 20° ~sec 20° is equal to (1988 -2 Marks) @ 2 () 2sin20°%5in 40" © 4 (@) 43in20°%sin40" ‘The general solution of| sin x~3 sin 2x + sin 3y= c0s.x~3 cos 2x + 60s 3xis (1989-2 Marks) © Se @ mF e+e ane 3 © cost (®) ann+cos 3 © cost @ 5 ‘The equation (cos p~1)3? + (cos p)x+ sin p= 0 Inthe varablex, has eal rons. Then pcan take any value in the interval (1990-2 Marks) ) @ Gx 22, [Number of solutions ofthe equation (1993 - 1 Mark) tan x+seex= 2cos lying the interval [0, 2] is @ ©2%) © x0 © ( @o ®t ©) 2 @ 3 Let dere then (sec2x—tan2x) equals. (1994) om) oath) (x3) (@ (x43) Let m be a positive integer such that sin eos = then (999 (@) 6sns8 (b) dense. (©) 40 onlywhen @>0 (b) <0 forallreal g (©) 20 forallreal 9 (@)_<0 onlywhen 0<0 fone sins = The number of distinct real roots of =Ointhe interval -F @1 @3 The maximum value of (os ,)(e084,)..60s a), under the restrictions 0sa,,0,, 15 F and (cota) (cota) (cota,)=1 is 2001s) @t en) @ 2b) 1m Ian fa+ B= ni2and += 4, then tan equals @) 2anB +tam) (@) tanB-+Htany (©) tanp + 2tany (@) Dan +tany The numberof integral values of k fr cosx+5 sinx= 24+ | asa solution is @4 68 © 1 Given both @ and 4 are acute angles and sin = 1 4 coxd= Len thevaluof0+ gbetngsio (2045) © (53) o 4] @ (24] cos(a.~ B)= 1 and cos(a.+ )= I/e where a, B € [~, x] Pairs of a, B which satisfy both the equations is/are (20035) @o ®t © 2 @ 4 ‘The values of 8 € (0, 2x) for which 2sin2—5 sind +2>0, are (2006- 3M, -1) o 2.3) GP_3480 28. Let oe(s. and f, = (tand)®, ¢, = (tan), = ca} and 1, (60) then (2006-301), @ natn © urh>h>h © t>arnrn @ apeoi>e 26, ‘Thenumber of solutions of the pair of equations’ 2rin® e020 =0 3200320 —3sind =0 4 inthe interval (0,27) s (2007-3 Marks) @ zo) one © two @) four 27. For x (0,8), the equation sine + 2sin 2x—sin’Sr= 3 has (EE Adv. 2014) (@) infinitely many solutions 28, (©) three solutions {6) one solution a (@ no solution vase frecna:ae0e$] se ametal dnt solutions ofthe equation 3 sec x + cosee x + 2(tan x = cotx)=Ointheset Sisequalto (JEE Ads, 2016) In a @ = % 4 ‘ sn © 0 oF 29, The value of isequal 4 to 2 (JEE Adv. 2016) @ © 2(3-V3) © @ 2(2-W8) 9. DD UCU keecra 1, {t+0082 146053214 e085 (1+ 0082 1 . 8 8 8 g) is caval os (1984-3 Marks) 1 x @ > (b) cos 1 1 © 5 @ OF 10. Theexesin3|snt(°E-2)ssntx+0) | ant{Z+a}esin'ts-a)] is equal to (1986-2 Marks) @ singe +0860 (©) none of these: ‘Thenmumbs ofall posible ipl (2,43) suh hata, + ay cons) asta) =O forall xis (1987-2 Mark) (a) zero (b) one (©) three: (d) infinite (¢) none ‘The values of @ lying between 6 = 0 and @= 1/2 and satisfying, the equation (1988-2 Marks) T+sin?0 cos? dsin4o sin?@ — L+e0s7@ 4sin40 | =0 are sin?@—cos?@ N+ 4sin 40 (@) 7nd (b) Sx (©) Nerd @) eA Let 2sin2x-+3sinx-2> 0 andx2—x-~2<0 (xismeasured in radians), Then x lies in the interval (1994) o (4) © 2 © (- off ‘The minimum value of the expression sina.+-sinB+siny, where a, B, 7 arereal numbers satisfying a+ B+ y= is (@) positive @) zer0 (1995) (©) negative @ 3 ‘The number of values ofx in the interval [0, Sx] satisfying, the equation 3 sin? x—7sinx+2=0is (1998-2 Marks) @ 0 ws © 6 @ 10 ‘Which ofthe following number(s) isare rational? (2998-2 Marks) () cos 15° (@)_ sin 15° cos 75° (1999-3 Marks) (@ sints° (©) sin 15° cos 15° Fora positive integer n, let 40) -(wm 8) et 0 0 00524)", Then w a(%) w a(S) oa @ «Ga © m4 1. For 0.<0-<, the solution (8) of M. For 0<0.<%, thesolution (m—Dx ms Y cosee (04) casee (4M) 43 is(are) 2009) ye = Ld, 4 oF oF OF OF 12. Let 0, € [0, 2x] be such that 2 cos0 (1 — sin @) = sin?0 (tan +cot cost tan(2n-0) >0and iano <5 om gammy an x ee @ Wee (b) ae fry 3 3x © Fe0cZ @ Beocre 13. The number of points in (20 0), for which x?—xsin x—cos: (JEE Adv. 2013) @ 6 ) 4 (o) 2 @ oO 14, Let fix) =x:sin mx, x>0. Then for all natural numbers n, f(x), vanishes at EE Adv. 2013) (@) unique point nthe interval (nn+5) $ Anige pints tna. (©) A.unique pointin the interval (n, n+ 1) (@Twopoints in the interval (a, n+ 1) B23 Subjective Problems tana= =" andi offa+B). (a) Draw the graph of y ant = 1 athe paslvaes (1978) (cnx +cos) fom =~ 0 2 = 4 sin (a—p)= 5, and a, B lies (b) eos (a+ 6) = $, sin (a—B)= Sand a, Bk between and 7 find tan2e (1979) Given + y=, provethat sin?a+ sin?B—sin?y=2 sina.sinB cosy (1980) Given t= {vss} and Fis)= cosx—x (1-43): find fA). (1980) Feral @in (0, x/2] show that, cos(sin9)> sin (050) (1981 - 4 Marks) 6 10, R 13. 4. 16. 1. 19. 20. trop ic-vise Solved Papers - MATHEMATICS. Without using tables, prove that i (1982-2 Marks) (sin 12°) (sin 48°) (sin 8 on ha) (1983 -2 Marks) Find all the solution of 4cos? xsin x 2sin? x =3sinx (1983 -2 Marks) Find the values of re(~x, + x) which satisfy the equation (1984-2 Marks) Provethat tana+2tan2a +4tanda +8 cot8a cot « (1988-2 Marks) gftleoss seo? ato) 2 gb ABC isatriangle such that sin(24-+B)= sin (C-A)=~sin(B+20)= + If 4, B and Care in arithmetic progression, determine the values of 4, Band C. (1990-5 Marks) Ifexp {(sin’e + sinty + sinés + co) In 2} satisfies the equation 2° 9x + 8 = 0, find the value of cose sexe F (1991 - 4 Marks) tanx. Tan3e Show that the value of “2° , wherever defined never lies berween + and 3. (1992 4 Marks) Determine the smallest positive value of x (in degrees) for which tan(x+ 100 an (x + 50°) tan(x) tan (x~ 50°), (1993 - 5 Marks) Find the smallest postive number p for which the equation cosp sin x)= sn(peos x has a solution x« [0.2] (1995-5 Marks) equation (1~tan6)(1 +tan6) sec? 0+ 210" (1996-2 Marks) sin:xeos3x Prove that the values ofthe function donot lie Sin3xe0sx berwoen 4 and 3 for any rea, (1997-5 Marks) 2k net Prove that _(n—k) cos where m 2 3 isan integer, (1997-§ Marks) In any triangle ABC, prove that (2000-3 Marks) A Bool A cB ren l cot + cot F-+o0t S = cot eat cot 2x + Sx? Find herangeofvalues oft fer whieh sine= <5 ve[-24] (2005 -2 Marks) GP_3480 Trigonometric Functions & Equations. ——__________» F Eto Ms DIRECTIONS (Q. 1): Each question contains siatements given in two columns, which have to be ‘matched. The statements in Column are labelled 4, B, Cand D, while the statements in Column-I are labelled p, q, rs and t. Any given statement in Colunn-I can have correct matching with ONE OR MORE statement(s) in Column-Il. The appropriate bubbles corresponding to the answers 10 these questions have to be darkened as illustrated in the following example Ifthe correct matches are A-p, s and t; B-q and r: C-p and q; and D-s then the correct darkening of bubbles wil look like the given. A B (| OOOO 2 3. Inthis questions there are entries in columns | and 2. Each entry in column 1 isrelated to exactly one entry in column 2. Write the correct letter from column 2 against the entry number in column | in your answer book, sin3a, 0320. * Column (A) positive @) negative Ba Integer Value Correct Type ‘The numberof all possible values of 8 where 0 <0 x, for whieh the system of equations (+) c08 30=(ay2) sin 30 0830, 2sin30 ? (72) sin 30 = (+22) c05 30 +ysin30 have solution (xy p.%) With yp Zp #0, 8 xsin30= e010) that 04% forn, 5 , 41,42 and tan O= cot 50 as well as sin 20 = cos 40 is ‘The maximum value ofthe expression e010) (1992 2.Marks) O) @ © ©) ‘Sin 0¥3sinDe0s0-r5e057 eo) 4. Two parallel chords ofa circle of radius 2 areata distance ‘5+1 apart Ifthe chords subtend at the center , angles of 5 ona 2 e010 FF 2010) {and =, where k> 0, then the value of] is [Note :[k] denotes the largest integer less than or equal tok] ‘The positive integer value of n> 3 satisfying the equation 1 1 6. The number of distint solutions of the equation ear 4 asin xcos® 5 J cost 2x+cost x-sin4 x4+-c0s®x4+sin® x= 2 4 intheinterval [0,2] is (EE Adv. 2015) The period of sin? 9 is 12002] @ 2 (b) © In @) 22 Thenumber ofsoluion oftan.-+ sec = oon (0,2 nis 12002] ®2 3 @0 @1 Which one is not periodic 12002) (@) | sindx sin? (0) cos VE + caste (©) 0s 4x + tan2x (@)_cos2x +sinx Let a,B be such that ABCD is a trapezium such that AB and CD are parallel and BC. CD. If ZADB=0, BC=pandCD=<, then AABisequl to HEE M2013) (Pg sino Png? coso © pemdasind © “eos0+gsind Pe (P+ )sin0 © Feosd+q?sind © (pees gsind) The expression HEA 5 £91 canbe writen as T-cotA tan A VEE M2013], (b) sec cosecA + 1 (@)_secA + cosecA, (@) sinAcosA+1 (©) tanA+cotA, Let fe (2)= Z[sint xcs) where xe R and 421 Then fa(2)~Fo(2) eauals [bre M2014) L 1 1 1 ®7F OF OF 3 1f 0x < 2x, then the numberof real values of x, which satisfy the equation o0s x cos 2x cos 3x cos 4x =i: IEEM 2016) @7 ©3 9 @s GP_3480 CHAPTER Complex Numbers I Secton-a 4 Rs Ifthe expression (4987-2 Marks) (ols) [ml] isreal, then the set ofall possible values of xis For any twocomplex numbers andb. Lae, =, and any realm (1988: ber a Marks) bey + | bey + Ia, b €, are the numbers between 0 and I such that the points 1,2, 1+ bland 7,=0 form an equilateral and b= (4989-2 Marks) ABCD isa tombs. Is diagonals AC and BD interset a the point M and satisfy BD = 24C. I the points D and M represent the complex numbers + iand 2-1 respectively, then A represents the complex number. (1993-7 Maris) Suppose ZZ Z,ate the vertices ofan equilateral angle inscribed inthe citle]=2, 182,214 WB then 2 on Za (1994-3 Marks) The vale ofthe expresion 19(2-<9)(2—e0?)+20(3-10)(3-? +-...+(=1).(n-o\(n—?), whee isan imaginary cube ooo unity (1996-2 Marks) 8’ RS For complex number =) = +i), and triangle, then a= ata, we sy if.) Sxy and 3 S yp. Then forall complex L with Iz, wehave = 0. (1981-2 Marks) Ifthe complex numbers, Z,,Z, and Z, represent the vertices ofan equilateral triangle such that IZI=1Z|=1Zplthen Z,+Z,+2Z, (1984-1 Mark) [fthree complex numbers are in A.P.then they lie on acircle inthe complex plane (1985-1 Mark) The cube roots of unity when represented on Argand. diagram form the vertices of an equilateral triangle (1988-1 Mark) (oll MCQs with One Correct Answer Ifthe cube roots of unity are 1, «, o?,then the roots of the equation (x~1)'+8=Oare (1979) (@) -1,1420,1420? — (b)~1,1-20,1-207 @ -N-1-1 (@) None of these ‘The smallest positive integer n for which (1980) () n=16 (@) none of these x-+ iy which satisfy theequation (4981 -2 Marks) (@) thexaxis (©) thestraight ine y=5 (©) acircle passing through the origin (@_ none of these ny 5) vthen (1982-2 Marks) ©) Im (@)_Re@)>0, 1m 2| represents the region given (1982-2 Marks) () Ro(e)<0 (@) none of these xy and @=(I~iz)/(2—H), then |o[=1 implies (1983-1 Mark) that, inthe complex plane, (@)_zlieson theimaginary axis (b) lieson the real axis (6) =lieson the unitcircle (@) None of these EBD_7202 oy 10, nL 2. 13. 4. 16. 1”. trop ic-vise Solved Papers - MATHEMATICS. The points 2,2 22 inthe complex plane are the vertices ofa parallelogram taken in order if and onlyif (1983-1 Mark) @ a ta72 +5 ©) at3=2 4% © 3+ (@) None of these fa, b,candu,v, ware complex numbers representing the vertices of two triangles such that 1=7)a+ rband w= (1 =P) ry, where risa complex ‘number, then the two triangles (1985-2 Marks) @ havethesamearea —(b)_aresimilar (©) are congruent (©) none of these Ifo (+ Ijisacuberootofunityand (140)? =4+Bo0 then Aand Bare respectively (19955) @o wi © 10 @ 11 Let and @ be two non zero complex numbers such that then equals (19985) © 3 @ -3 snd « be two complex numbers such that =| < 1, | o |< Land|2+io |=|2~10|=2then zequals (1998S) (@ Lori) fori © Lor-1 @) for1 For postive integers n,, nthe value ofthe expression CDM HC HPY" HLA LY where = Jp {sa eal numberifand onlyif (1996-1 Marks) @ nant ©) manyat © many @ n> 0.20 Ca way oC way Wis YT then 4 +5 [=~ 48) Bb 8 is equal to (1999 -2 Marks) @ 1-8 ® -1NIO WI @ -WF arg(e)<0, then arg 2) -arg(2) 2omas) @r © oF oF Izy, z,and.,arecomplex numbers such that (2000) A (@) equaltol (©) essthan 1 (©) greater than 3 (@) cqualto3 Lets, and =, be ni roots of unity which subtend a right angle atthe origin. Then n mustbe ofthe form (20015) @ dhl () 442 ©) ARH (@) dk ‘The complex numbers =, , and 2, satisfying |. then [21 +22 + 25s are the vertices ofa triangle which is Fa-85 (@) ofareazero (0) right-angled isosceles (©) equilateral (©) obtuse-angled isosceles (2001s) 19. 20, a. 2. 23. 4, For all complex numbers =), 2, satisfying [=|+12 and |z;34/=5,theminimumvalieotls-sii8 (20028) @ 0 (b) 2 ©7 @ 7 1h £54 (where = ¥ =D, then Reis @ 0 (2003S) © lalpaP Ife (¢ 1) bea cube root of unity and (1 + o#)"= (1+ 0", then the least positive value of n is (20048) @2 w3 os @6 ‘The locus of = which lies in shaded region (excluding the boundaries) is best represented by (20058) (@) 22/54 11>2and arg (e+ 2and|arg (e-1)] 18 Let @ = 5 +175, then the value ofthe det. (2002 - 2 Marks) ©) 30(-1) (@ 30(1-0) is purely real wherew= a+ if, Bx Oand = 1, then the set ofthe values of is @ teH1 © G41 (2006 - 3M, -1) GP_3480 Complex Numbers 25, 26, 2. 28, 29, 30. 3. ‘A man walks a distance of 3 units from the origin towards the north-east (N 45° E) direction. From there, he walks @ distance of 4 units towards the north-west (N 45° W) direction to reach a point P. Then the position of P in the Argand plane is (2007-3 marks) (@) 344i ©) B-4pe" © (+3e @ G40" itl and = #1, then all the values of (@)_aline not passing through the origin (2007-3 marks) (b) 2 (© thexaxis © theyeaxis [ApaticlePtart from the pointy =1-+2i, where i= JI moves horizontally avay ftom origin by S units and then ‘vertically away from origin by 3 units foreach a pont = From; the particle moves /Z units in the direction ofthe vector j+ and then it moves through an angle FI in anticlockwise direction on a circle with centre at origin, to The point, is given by 2008) ©) 7461 @ 6471 1s 08 8 + isin &. Then the value of > Im = Pry 4 at O=2%is 2009) 1 1 1 1 sana © einae O aaae Letz=x+ iy bea complex number where: and yare integers. ‘Then the area of the rectangle whose vertices are the roots @ ofthe equation: 22° +22) = 350s ew & @ & (b) 22 ©) 0 @ » Letbea complex number such tat the imaginary part of: isnon-zeroanda=2 ++ 1 isreal, Then a cannttake the value 012) 1 1 1 3 @- %3 O77 WG Letcomplesnumbersaand *licencirdes(x-xg? +(y-¥o)? =F and (x — x9)? + (y—¥o)?= 47 respectively. If %5=%p * i sais the equation 2zgl? =r? +2, then| (JEE Adv. 2013) 1 1 oF > OF M3 1 MCQs with One or More than One Correct It, a+ib and 2) =c+id are complex numbers such and Rete, -+ic and vy =b+id satisfies — (1985 -2 Marks) ©) bel (@ none of these then the pair of complex @ © Let = and =, be complex numbers such that =; # => and. a Re(mifa) =0 at imaginary part, then = a 2 |. fz has postive el part and =, has negative 2 ‘maybe 2 (b) real and positive (8) patlyimeginary (1986-2 Marks) (@) 210 (©) tealand negative (©) none of these. If; and 2 are two nonzero complex numbers such that | 2, H12i1 +123 then Arg =~ Arg =) is equal to (1987-2 Marks) (@) -2 = @o0 @ = ® » = © oF © & in 28h _ ggg 2 The valueot 5) (sin — feos 24) is (1987-2.Marks) @-1 0 @-i @i (©) None Ife isan imaginary cube root of unity, then (1 +@- 0°)" equals (1998-2 Marks) (@) 180) 1280 (6) 128u? —(@) 128" B Thevalueofthesum (P+), wherei= J] equals (1998-2 Marks) @i Ort OA @0 ln v1 ia +19 then (1998-2 Marks) po 3 3 (@) x=3, (© x=, Let 2, and z, be two distinct complex numbers and let 2=(11).2, for some real number t with O-<1< 1 IfArg, (») denotes the principal argument of a non-zero complex ‘number w, then 2010) (@ [2-31+12-2, ) Arg anf —2))= Arg ( (@ Arg @-2)=Are(e, 10, 2,3,..). Further Hy = (some sot ofall complex numbers. If 2) € PAH, — 12 €PoH, and represents the origin, then Z2,0:)= (JEE Ads. 2013) x 5 an se ®F oF OF OF Leta, be Randa'+b # 0. atibr AAT. Ifa=x+ iyand2 ©, then (x,y) lies on (EE Adv. 2016) — oa ) 1 (Eo Jy andeentre (3 (e @ bao 1 (©) the circle with radius—3— and centre a<0,b#0 (©) thexaxis fora #0,b=0 (@_they-axisfora=0,b#0 5 RSS Express intheforms-iy. (1978) Toe0s0+2isind Mx=a+b,y =ay* OB and: {B+ by where y and parethe complex cube roots of unity, show thatx)== a+ (1978) ofa? far +8 Itx+iy= [28, ie 97) Find here auesofxand for which he iloning equation is satisfied 1 *— 24 Coop Let the complex number =,» and =, be the vertices ofan equilateral triangle. Let=bethe circumcentre ofthe triangle. Then prove that z 352. (1981-4 Marks) Prove thatthe complex numbers, 2 andthe origin form an equilateral triangle onlyit ait Ita, (ay 1 (1980) (1983 -3 Marks) 14y_ arethen roots of unity, then show that )(1~a3) 1 ~4,_,) = (1984 - 2 Marks) 10, 2 1B. 16. 1”. 19. trop ic-vise Solved Papers - MATHEMATICS. ‘Show that the area ofthe triangle on the Argand diagram formed by the complex numbers =, izand = + ist iis Hel (1986 - 2% Marks) Let Z,= 10+ 61and Z,=4 +61. If sany complex number (2-2) such that the argument of (Z-2) is & then prove that |Z-7~91)= 3y2. (1990 - 4 Marks) fis +2—2+/=0, then show that|z|= 1 (1995 § Marks) IFIZIS1.IWS1, show that |Z-WP s(Z|-[W Dp? Arg Z— Are? (1995-5 Marks) Findall non-zero complex numbers Zsatistying Z =i2 (1996-2 Marks) Let, and =, be rots ofthe equation *¥p2+q=0, where the covfiientsp andq maybe complex numbers. Let and B nd, in thecomplex plane. Z40B =a +0 and 04 = OB, where Ois he origin, prove that p-wee( (1997--$ Marks) For complex numbers and w, prove that wsinf?= ifand onlyit== wor (1999 - 10 Marks) Leta complex number a, #1, bea root ofthe equation #°9-2 -21+1=0, wherep.q are distinct primes. Show that cither I+a+o2+...to'=0orl+atat...tat"!=0, but not both together. (2002-5 Marks) Iz and =, are two complex numbers such tat << then prove that (2003-2 Marks) Prove that there exists no complex number = such that le1, Let @=e3 , anda, b,c x,y; zbenon-zero complex numbers (@) 25nd29 (b) 30and34 such that 2011) (© 35and39 (@ Handss atbtenx 3. Let z be any point AN BAC and let w be any point ate ieee satisfying |w~2~i| <3. Then, le|~ [w+ 3 lies between « (@) Gand3 () Bando 2008) Then the vatue of ZR *L7F lel, © ands (® Sando laF +16 +1eF 7) ke passace2 3. Forany integer let a, ers(*) + sin“), where Let $= 5, 08,955, where 2 Bi . Dlera-ael Sy teee|e)<4, 5;0} aoe 4. ArcaofS= (EE Ads. 2013) is (IEE Adv. 2015) toe 20m 16s Mn oF oF oF oF If Section pA 33 and waretwononzero complex mumberssuchthat|=/=|vl§, Let 2, and Zy be two roots ofthe equation and Argz-+ Argw m then zequals 12002) eneeeata nan hat Oia eee ing complex, Furr ase tt 2. fle—4|<|2~2, itssoation is given by 12002) and Z forman equilateral triangle Then (@) Re@)>0 (b) Re(z)<0 {2003} (©) Re(e)>3 (@ Rele)>2 1b A) a? =3 3. Thelocusofthecente ofa circle which touches the circle z-z|=aand|2—z,|=bextemally 2, & 2 ate 6 12003) ‘complex numbers) will be 12002) (a) ancllipse (b) a hyperbola (a) x=2n4+1, where nis any positive integer © acicle (@) none of these (©) x=4n , wheren isany positive integer 4. Ifzando are to non-zero complex numbers such that (¢) x= 2n where nis any postive integer © then zs (@) x= 4n-r1, wherenis any postive integer, Vand Arg(=)~ Arg(o) hen Zo isequalto 7 et zand wbe complex numbers such that and 2003] argsw= x. Then argequals 12004) ®1 @-1 @F sn x 3x x ®F OF OF @F GP_3480 Complex Numbers 8 10, R 1B. 4 16. ; aesociyand Sprig ma (E02) ft ti equal to 12004) @2 41 ©2 @t =1fe12P? +1, thenz lies on 1204) (@) ancllipse (©) theimaginary axis (© acircle (@)therealaxis Ihe eube roots of unity are 1, ©, «? then the rots ofthe equation (x=? + [2005] @ +, @) -1- © -1,1-20,1-207 @ -1,1420,14207 If 5 and 22 aretwo.non- zero complex numbers such that 1+ 1221, thenarg arg 2; isequalto 12005] @-2 @o @ ‘and | © |= 1, then z ies on [2005], (@)anellipse (©) astraight line (b) acirele (@) aparabola 23), pw rhea (sin? ics @ i (bt -1 @-i If :? +241 =0, wherezis complex number, then the value Ms 12006] @ 8 ws @6 WR If | z+ 4] < 3, then the maximum value of [+I is 12007] @ 6 0 ws @ 0 ‘The conjugate of a complex number is — then that complex number is 12008 1 om o @ ra i v. 19. 20. 1 2. 2B. Let & be the rel line. Consider the following subsets ute plane Rx: S=(ax,y)-y=x+ Land 0 (@) Tieinthe interval (1,2) Mls 2. GP_3480 trop ic-vise Solved Papers - MATHEMATICS. ‘A complex number z is said to be unimodular iff 2-22 Suppose 2, and z, are complex numbers such that 5 is unimodular and 2) is not unimodular. Then the point 2, Tieson a VEE M2015] [SEE M2016} sio!( @ circle ofradius2 CO) (%) ©) circle ofradius J3 (©) straight Line parallel tox-axis ot oe (@ straight line parallel to y-axis. 3 6 243isind T-2isind 25. Avalueof 8 for which is purely imaginary, is: CHAPTER Quadratic Equation and Inequations (Inequalities) | cy JEE Advanced/ liT-JEE 4 Rs The coefficient of” in the polynomial (1) @—2) = 100) is (1982-2 Marks) 2. If 241V5 isaroot ofthe equation 3 + px+q=0, where pand q arereal, then (p, q)=(_ . ) (1983-7 Marks) 3. Ifthe product ofthe roots ofthe equation “3la+2 eM T= Os 7, then the roots are eal for k (1984-2 Marks) 4. tbe quadratic equations + ax-+b=Oands?+br-+a=0 (2+) havea common oo, then the numerical value of a+bis (1986-2 Marks) logy ons (V3 +5) =0 is (1986-2 Marks) The solution of equation 6 tte I, the inequality (n}!"" <"=" poids, (1981-2 Marks) 2.‘ Theequation 2x? + 3x-+1 =O hasan irrational rot. (1983-1 Mark) 3. Hfa0,6>0and.c> 0. Then the roots of the equation axe+bx+c=0 1979) (@) arereal andnegative — (b) havenegative real parts (©) both (a) and (b) (@)_ none of these Both the roots ofthe equation (eB) Gc) + (xa) (eC) + (2a) (x—B)=Oare always. (@)_ positive () real (1980) ()_ negative (@)_none of these, The least value of the expression 2 logge —log,(0.01), for x> Lis (1980) @ 10 & 2 (©) -001 (@)_none of these. IG? px Disafactorof(ar+bx-+e),then (1980) @) @+e-ab ©) @ ab © @-e=ab (@) none of these EBD_7202 Pag 8 10, 2 13. 4 16. 1". The number of real solutions of the equation [xP-3|x]+2=0is (1982 -2 Marks) @4 M1 @3 @ 2 ‘Twotowns A and B are 6 km apart. school isto bebuilt to serve 150 students in town A and 50 students in town B. IF the total distance to be travelled by all 200 students is to be as small as possible, then the school should be built at (1982-2 Marks) (@) 45km from town (@) 4SkmtiomtownB (@®) town B (©) town fp, q,rare any real numbers, then (@)_ max (p,q)0is (1982-2 Marks) @ -4=1-—> has (1984-2 Marks) (@) no root (©) one root (6) two equal roots (@) infinitely many roots Ia? 62+ c2= 1, then ab + be + ca les inthe interval (1984-2 Marks) 1 1 1 @ 52 642 © F50@ F453 flogg 3 (%— 1) 4 ‘Ifaand B (a.a, then the equation (x—a)(x—b)—I (@) both roots in (a 6) (6) bath roots in 2,2 (c) both roots in (b, +2) (@)_ one root in (20, a) and the other in (b, +20) For the equation 3x? px-+3=0,p>0,ifone ofthe rootis has (20008) square of the other, then pis equal to 20005) @ 13 (b) 1 @ 3 @ 23 1,4. a postive real numbers whose products @ fixed number then theminimum value of Oy 43+ sooty +20, i8 (20028) (@) ne)" (bd) (tcl* (©) 2ncl™ (d)_ (#1)Q2c)"" ‘The set ofall real numbers x for which x?7—|x+2|+x>0, is (20028) @ Ce-2)0Q2) — ) Fa-v2)u(i2.«) © Caiutie) — @ Wx) te a (0.3) then VP ox s always grater than oF equal to 20035) @ 2tana 1 © 2 @ sa GP_3480 Quadratic Equation and Inequations (Inequalities), —_______________» ot 30, Forall‘x’,x2+2ar+10-3a>0, then the intervalin which 37. A value of for which the equations ‘a’ lies is (20048) w+ bx-1=0 (@) a<-5 (b) -SS @) 2 By and > Ba, then Ie a a a + B, equals (JEE Adv. 2016) Soy + ti —qxtre (a) 2 (sec @-tan 0) (b) 2secO J 2B betheroots ofthe equation x*— gx +r=0, Then the eee ot value of ris (2007-3 marks) 2 2 Bs MCQs with One or More than One Correct ©) 5(P-92a-P) —) 5 9-PX2P-9) (x ayix-8) 2 2 For ral x, the function =="? i assume al real © Fle-2X2a-p)— @) $OP-aK2q~P) we values provided (1984-3 Marks) 34. Let pand q be real numbers such that p#0,p°#q and —*@) a> b>e ©) axbc>b @) a +g. Ifaand arenonzero complex numbers satisfying. oe-1 ceeB=_panda’+f'=q.thenaquaiceqation having _Sisthesetofallrel such tht —7*_— pase, and Bas its rots is ao) then $ contains (1986 -2 Marks) 6 3 301 3 3 ® (-* 3) oy (- 4) ) (PP +g —(p +2a)x4 (p> +4)=0 7 7 Ba, -( = +(p+g= ©) (+ a)x? —-(p -2a)x+(p +g) =0 © (- @ (2.3) ©) (p>—a)x? -(Sp? -2g)x+(p)-4)=0 © (a)? 5p) 29x (pa) Ort (8) (p' -@x? Sp? +2g)x+(p? -q)=0 ‘fa, band care distinct postive numbers, then the expression 35. Let (3) be the solution ofthe following equations (b+ c~aye+a~bya+b~c)~abeis (1986-2 Marks) pt on (@) positive (b) negative 2x = Gy)" (©) non-positive (@)_ non-negative er aby (©) none of these a=? Ifa, b, c,d and p are distinct real numbers such that ‘Then xy is eo (2+ B+ yp 2 (ab+ be + cdip+ (+2 +2) <0 then , bye, d (1987-2 Marks) 1 1 1 (@ areinA.P () arcinGe OF WF OF @ 6 (©) arein HP. (@)_ satisfy ab = ed 36, Let a and fi be the roots of x? ~ 6x ~2=0, with a> B. If (©) satisfy none of these . “ay ~2a “The equation os *? +182 #-5/4 = J has ~B" for 2 1, thea the value of ss (a) at least one real solution (1989 - 2 Marks) (b) exactly three solutions ony (c) exactly one irrational solution @l (b) 2 ©3 @4 © at ee (@ complex roots cary = tt poet ops nabs iy Then um : ‘irs 2a) ( apssveinege: —@) dihleoyn © euatwne! —— @ aewrlastann [Number of divisor ofthe form 4n-+2(n20) of theinteger 240 is (1998 - 2 Marks) @4 m8 © 0 ws I= 4", then x= (EE Adv. 2013) 2logs2 2 © Flogs2=i ©) 2tog3 1 2log 3 © T0453 Blog: 3-1 Let Sete st ofl none rel nunbesaschtat he tuedutaqatin arama tre dit ea ts SMdssoanhngte inequity eet Whe ilies ar) wot e857 (OBE Aa. 2015) o ( o ( (1978) = (1978) a-x") ‘where m and n are positive integers (n < m), show that (m,n 1)= (m= Ln 1) +x" "(m= 1m. SoWveforx: Yeri-VE=T=1 as) Salve the allowing equation F< a9) 2log,a+logga+3 log», a=0,a>0 Show thatthe square of 25-1543 ie rational Ssy2 -V38+5V3 suber a7) Sketch the solution set of the following system of inequalities: w+ y22e 2 0;3u-y-12 $ O;y-x £ 0;y2 0. (1978) Find all integers x for which (1978) (x-1)<(@+ IF <(x-3). a ace the roots of? + px +q = 0 and y, 8 ate the roots ofs? + x50, evaluate (0-7) (a~8)(B-1) (B—8)interms ofp, a, rands. DDeduce the condition that the equations have a common root 979) 9. 10, Ra, 3B. 14, 16. n. 19. 20. 24 2. top ic-wise Solved Papers - MATHEMATICS Given n'< 10" fora fixed positive integer n > 2, prove that (n+ 1)8< 10%! cass) Lays (4980) V @-3 Fin athe el vues of fr which yaks rea ales For what vals of m, des the tem of uations hhas solution satisfying the conditions.x>0,y>0. (1980) Find the solution set of the system (1980) xe+2ytz=1; 2x—3y—w=2; x2 0;y20;220,w20, Show that the equation e"* has no real solution, (1982 -2 Marks) ‘mn squares ofeugal size are arranged to froma rectangle of dimension mby n, wherem and n are natural numbers. 1Wo squares will be called “neighbours they have exactly one ‘common side, A natural number is written in each square such thatthe number written in any square isthe arithmetic ‘mean ofthe numbers written in its neighbouring squares. ‘Show that this is possible only if all the numbers used are equal (1982 -$ Marks) one root of the quadratic equation ax? + bx += O's equal to the n-th power of the other, then show that L 1 (act) s(a" ort b= 0 (1983-2 Marks) Find all real values of x which satisfy ? 34.42 >0 and xi -2x-450 (1983-2 Marks) Solve forx; (5+2V6)""> +(5—2¥6)"" =10 (1985-5 Marks) For @ <0, determine all real roots of the equation x? — 2a| x a|-3a" (1986 - § Marks) ax 1 (Qt 45x42) HD (1987 - 3 Marks) Solve [x2 ¢4r43/ 42095 (1988-5 Marks) Leta, bcberel a+ br+¢=Ohas wore ro and C Find the set ofall x for which <0. B,, where a.<—1 and > 1, thenshow that 1+ $+ (1995 - 5 Marks) Let Sbe a squate of unit area. Consider any quadrilateral which has one vertex on each side of S. Ifa, bc, and d denote the lengths of the sides of the quadrilateral, prove that 2< abbr, (1997--§ Marks) GP_3480 pera cane eeraneare a od If GB are the roots of ax? + bx +6 = 0, (+0) and +8, B+8 aretheroots of 4x? + Bx +C=0, (40) for Pdac _ BP-4AC @ (2000 -4 Marks) 24, Leta, ebereal numbers with a0 and lt, be the roots ofthe equation ax? + dx + ¢= 0, Express the roots of ax? +abex+c)=Ointerms ofa, B. (2001-4 Marks) 25, Ifx?+(a~b)x+(1-a-b)=Owherea, b € Rthen findthe values of for which equation has unequal real root forall values ofb (2003 -4 Marks) 26. Ifa, b, ¢ are positive real numbers. Then prove that (a+? +1)) (er! > Tate et — (2004-4 Marks) some constant & , then prove that 27. Leta and bbe the roots ofthe equation x? 10cx—I1d=0 and those of x? - 10ax— 11b=0 are c,d then the valuc of atb+c+d,whenaebeced,is (2006- 6M) H ECS 1. Leta, b,c,p,qbereal numbers. Suppose , Bare the roots, 1 ofthe equation x2 + 2px-+ = Oand a, F are the roots of the equation ax? + 2bx +e 0, where ? ¢ {-1, 0, 1} I Sections #7FFy 1. Ifa. Bbuta?=Sa—3and f= 5-3 then the equation having a/P and Pat as its roots is 12002] @ 3-1843=0— (b) 3+ 19e-3=0 (© 3P-1%-3=0 (@) x ~-Sr+3=0. 2. Difference between the corresponding rons of? ax+b-0 and +br+a=0 is sameand at» b, then 12002] (@ at6+ 0) atb— (© a-b-4=0 (@ a-b+4~0 3. Product of real roots of the equation x*#x}*9=0 [2002], (@) is always positive (b) is always negative (©) does not exist (@) none of these 4. Mpandg are the roots of the equation °4p¥q=0, then @ p ©) pel [2002] © p © p=-2.¢1 5. Ifo,b, care distinct +vereal numbers anda +2?+c?1 then ab +be+cais (2002) (a) less than 1 (©) equal to 1 (©) greater than | (@)_anyreal no. 6 If the sum of the roots of the quadratic equation, ax? +br-+e=0 is equal to the sum of the squares of their ab ye reciprocals, then ©, "and arein 12003] (a) Arithmetic - Geometric Progression (b) Arithmetic Progression / @IEEE 10, STATEMENT -1: (pq) (6?—ae) > 0 and STATEMENT-2: b pa or ¢# ga 2008) (@) STATEMENT - 1 is True, STATEMENT - 2 is True; STATEMENT - 2 is a correct explanation for STATEMENT: | (6) STATEMENT - | is True, STATEMENT - 2 is True; STATEMENT - 2 is NOT a correct explanation for STATEMENT-1 (©) STATEMENT- 1 is True, STATEMENT -2is False (@_STATEMENT- 1 is False, STATEMENT - 2s True $4 Integer Value Correct Type Let (x, 9,2) be points with integer coordinates satisfying the system of homogeneous equations A3x+2y+2 Then the number of such points for which xisyi+2 S100is 2009) ‘The smallest value of f, for which both the roots ofthe equation 28k + 162k 1)=0 are real, distinct and have values atleast is (2009), ‘The minimum value of the sum of eal numbers a5, a4, 30,1, a%and a! where a> Ois eon) ‘The number of distinct real roots of at dy} + 12x +x-1=0is ony (©) Geometric Progression (@) Harmonic Progression ‘Theva oft far whichone root of the quadratic equation (a? ~Sa+3)x? +Ba—1)x+2 is twice as large as the other is 12003) aL 5 aa a @-3 &F ©-3 3 ‘The numberof ral solutions ofthe equation x? -3h]42=0 is (@3 (b) 2 (4 @t ‘The eal number x when added to its inverse gives the ‘minimum value of the sum at x equal to 12003} (a) 2 (b) 2 1 @-l Let two numbers have arithmetic mean 9 and geometric ‘mean 4, Then these numbers ae the roots ofthe quadratic equation [2004] @ 2-18-1620)

3 (©) -l I, thenthevalueof Tay ienalto [JEE M 2015}, @ 3 &) -3 © 6 @ -6 The sum of all real values of x satisfying the equation (25x45) 21 ig HEE M2016) @ 6 5 (©) 3 (@) -4 GP_3480 CHAPTER Permutations and Combinations I Section-a MEISE WIRES 4’ BE 1. Ina certain test, a; students gave wrong answers fo atleast ‘questions, where = 1,2, .., Nostudent gave more than K wrong answers, The total number of wrong answers sivenis (1982-2 Marks) 2. ‘Theside AB, BCand C4 of triangle ABC have 3, 4 and 5 interior points respectively on them, The number of ranges that can be constructed using these interior points as vertoesis (1984-2 Marks) 3. Total number of ways in which ix “Mand four “signs can bearranged in aline such that notwo signs oocur together is (2988-2 Marks) 4. There are four balls of different colours and four boxes of colours, same as those of the balls. The number of ways in ‘hich the balls, oe each in a box, could be placed such that ball does not goto a box fits own colour is 3’ Ro 1. Theproduet of any rconsecutive natural numbers is always divisible by! (1985-1 Mark) [el MCQs with One Correct Answer 1. *C,.)=36,"C,=84and"C, ,)=126,thenris: (1979) @ 2 © 3 (@) None of these. 2. Ten diferent leters ofan alphabet are given. Words with five letters are formed fom these given letters. Then the ‘number of words which have at least one lter repeated are (1982-2 Marks) ©) 320 (@) none of these _ 3. Thevalucofthe expression *7C, +574, isequalto rl (1982-2 Marks) &) (@)_none ofthese 4. Eightchirsare numbered It 8, Two women and threemen wish to occupy one chair each. First the women choose the chairs rom amongst the chairs marked 1 to4 and then the ‘men select the chairs from amongst the remaining. The number of possible arrangements is (1982-2 Marks) (@) 6760 (©) 9788 10, 1" 2 @ Sqx4cy Oma © 444% (@)_none of these A five-digit numbers divisible by 3 is to be formed using the numerals 0, 1,2,3, 4 and 5, without repetition, The total hhumber of ways this can be done is (1989-2 Marks) @ 26 6 w © @ @ 3s ow many different nine digit numbers can be forme from the number 223355888 by rearranging its digits otha the cad digits cocupy even positions ? 20008) @6 0% © Ww Let T, denote the numberof tiangles which ean be formed using the vertices of a regular polygon of n sides. If Ty y= Ty 21, then n equals 20018) B's" @ 7 Os @4 The number of arrangements of the letters ofthe word BANANA in which the two's do ot appear adjacently (20028) @ 0 & 0 © 9 @ 10 A rectangle with sides of length (2m ~ 1) and (2n— 1) units is divided into squares of unit length by drawing, parallel lines as shown in the diagram, then the number of rectangles possible with odd side lengths is (2008S) (@) (m+n? @) ann (©) mt (a) m(m+ n(n 1) Ifthe LCM ofp, gis P°¢'s?, where r,s, care prime numbers and p,q are the positive integers then the numberof ordered pair(p, is (2006 - 3M, -1) @ 2 © M © 2 @xB ‘The leters of the word COCHIN are permuted and all the permutations are arranged in an alphabetical order as in an English dictionary. The numberof words that appear before the word COCHIN is (2007-3 marks) @ mM OM O% WH The number of seven digiti_ntegers, with sum of the digits equal to 10 and formed by using the digits 1,2 and 3 only is (2009) @S © 6 on @8 EBD_7202 3. Thetotal number of ways in which S balls of different colours can be distributed among 3 persons so that each person gets at least one ball is (2012) @ 5 0) 10 © 20 @ 2 ix cards and six envelopes are numbered 1,2,3,4, 5, and. cards are to be placed in envelopes so that each envelope contains exactly one card and no card is placed in the envelope bearing the same number and moreover the card ‘numbered 1 is always placed in envelope numbered 2. Then (JEE Adv: 2014) @ 4 6 6 OB We ‘A debate club consists of 6 girls and 4 boys, A team of 4 ‘members is to be selected from this club including the selection ofa captain (from among these 4 memoers) for the team, Ifthe team has to include at most one boy, then the number of ways of selecting the team is (JEE Adv. 2016) @ % 6 0D © MW WS D TR eetencs An tedigit number isa positive number with exactly n digits Nine hundred distinct n- digit numbers are to be formed. using only the three digits 2, Sand 7. The smallest value of 1 for which this is possible is (1998-2 Marks) M6 7 @9 the number of ways it can be done is os 5 BOS Six X's have to be placed in the squares of figure below in such a way that each row contains atleast one X. In how ‘many different ways can this be done. (1978) F Eton tp 2 -awise Solved Papers - MATHEMATICS Five balls of different colours are tobe placed in there boxes of different size. Each box can hold all five. In how many different ways can we place the balls sothat no box remains empty’? (1981-4 Marks) _mmen andn women are tobe seated in arow so that no two ‘women sit together, Ifm> 7, then show that the number of mins! ‘ways in which they can be seated isp 1) (1983 -2 Marks) 7 relatives of a man comprises 4 ladies and3 gentlemen; his wife has also 7 relatives; 3 of them are ladies and 4 gentlemen. In how many ways can they invite a dinner party of 3 ladies and 3 gentlemen so that there are 3 of ‘man’srelatives and 3 ofthe wife's relatives? (1985 - 5 Marks) ‘A.box contains two white balls, three black balls and four red balls. In ow many ways can three balls be drawn from the box if atleast one black ball isto be included in the draw? (1986 - 21% Marks) ghteen guests have to be seated, half on each side of a long table. Four particular guests desire to sit on one particular side and three others onthe other sie. Determine the number of ways in which the sitting arrangements can bemade, (1991-4 Marks) ‘Acommitte of 12s tobe formed from 9 women and men, In how many ways this can be done if atleast five women have tobe included in a committee? In how many ofthese committees (1994 4 Marks) (@) The women arein majority? (©) Themen arein majority? oy my” (ner). (2004-2 Marks) If total number of runs scored in m matches is Prove by permutation or otherwise ‘an integer the k'* match are given by k. 2** 4, where 1 < k ) 210 @ 8S @ 1110 ‘Theset $= {1, 2,3, ....5 2} istobe partitioned into three sets A, B, C of equal size. Thus 4 U BU C= 5, AQB=BAC=ANC=6. The number of ways to partition Sis 120071 @ 2 aa 12 a ay 3a) sua Ina shop there are five types of ice-ereams available. A child buys six icecream Statement-1 : The numberof diferent ways the child can buy thesix cecreamsis "C, Statement 2 The number of different ways the child can buy the six ice-reams is equal tothe numberof diferent ‘waysof aranging 64°sand 4 sina row. 12008) @) Statement-1 is false, Statement? istrue (©) Statement-1istrue, Statement-2is true; Statement-2is correct explanation fr Statement (© Statement-1istru, Statement-2is true, Statement-2 isnot acorret explanation for Statement-1 (@ Statement-1 istru, tatement-2is alse How many different words can be formed by umbling the letters in the word MISSISSIPPI in which no two S are adjacent? @ 867, ©) 67.5, © 68."C, @) 7.5, Cy From 6 ferent novels and3 different dctonares,4 novels and dictionary areto be selected and arranged ina row on a shelfso thatthe ditionary is always inthe middle. Then the number of such arrangement is: 12009) (@) atleast $00 but less than 750 (©) atleast 750 but less than 1000 (©) atteast 1000 (@ lessthan 500 16. 1 19. 20, a. 2, ‘There aretwo urns. Urn A has 3 distinct red balls and urn B thas 9 distinct blue balls. From each urn two balls are taken cut at random and then transferred tothe other. The number cof ways in which this can be done is 2010] @ % 0b) 6 © 8 @3 Statement-1: The number of ways of distributing 10 ‘identical balls in 4 distinct boxes such that no box is empty isC, ‘Statement-2: The number of ways ofchoosing any 3 places rom 9 different placesis °C, bon] (@)_Statement-1is tue, Statement: istrue; Statement-2is, nota correct explanation for Statement-1 (6) Statement-1is true, Statement-s false (©). Statement-1is false, Statement.2 istrue (@ Statement is tue, Statement-2 struc; Statement-2is, correct explanation for Statement-1 Theseare 10 points in a plane, out ofthese 6 are collinear, if Nis the number of triangles formed by joining these points, then 12012) (@ n<100 ©) 100190 Assuring the balls to be identical except for difference in colours, the number of ways in which one or more balls ean be selected ftom 10 white, 9 green and 7black balls is (2012) @ 8 6 @ ~] oo ds Let, be the number of all possible triangles formed by joining vertices of an n-sided regular polygon. IT —T, = 10, then the value of nis. [EEM 2013} @7 5 © 0 @s ‘The mumber ofintegers greater than 6,000 that canbe formed, using the digits 3,5, 6,7 and 8, without repetition, is VEE M2015] @ 2% o 2 © 26 @ 12 ‘all the words (with or without meaning) having five leters, formed using the letters ofthe word SMALL and arranged asin a dictionary; then the position of the word SMALL is, VEE M2016) (@ som © 46% (o) 58m @ so GP_3480 CHAPTER Mathematical Induction and Binomial Theorem : rr JEE €dvanced/ iT: TIEE Green) ‘The larger of 99% + 100 and 101s. (1982-2 Marks) Thesum of the coeficients of the plynomial (1-+x~ 3x22 is (1982-2 Marks) Wax y= 14804242... thena=....andn= (1983-2 Marks) Let m be positive integer, Ifthe coefficients of 2nd, 3ed, and 4th terms in the expansion of (I +x) are in A.P, then the value of nis, (1994-2 Marks) The sum of the rational terms in the expansion of (2435)! is (1997-2 Marks) c RIeiireirremoraes Given postive integers > ,n>2 and that the eneffcient of Grit and (r+ 2} terms in the binomial expansion of (12) are equal Then (1983-1 Mark) @ n= @ m=2re1 © n= (@) none ofthese fe 3 The coefficient of x‘ in a-z) is (1983-1 Mark) 40s sos © Fe 3 430 © is @) none ofthese F ns ns The expression |" @e mye [0 | isa polynomial of degree (1992-2 Marks) @) 6 ) 9 @ 2 @ 4 rerreren (oal,"Ja(t)= ames (n+l) (nD (n+2 (n+2 o (Jo C5) © Ce (7) 2 Inthebinomial expansion of (a -6)n2 5, the sum ofthe S* and 6" terms is zero, Then aib equals (20018) (@) (n-sy6 @ rays © Sta-8 (@ st=5) (10) 20 ) (’) are sum S()(0° ). cvme(”) = rma) is ‘maximum when mis 2002s) @ 5 & 0 © 1S @ 20 Coefficient of in (142 (14e)(1 +i (20035) @) P+) BGT ©) BC, @ BC+ IE™IC,5 (2-3) "C0, then ke 20015)" @ Ca210) BA — © A313 @) 64.2) The value of (30/30) (30730) (30/30) (3030 Co faol{r has}*C2 ua} *Laoao) ise (2003s) 30" (30) (60 31) @ (i) © is} © (30) © loo) Fore=0,1,.. 10,4, 8nd C, dents, respectively, the coefficient of xin the expansions of (1 +x)"°, (2010) ww (1+xP and (1+2?°. Then LAtb8, yA, sequal to @) By-Co (0) Ayg(B Cy) © 0 © Cy-Bip Coefficient of x!" in the expansion of (e280 +27 +x) Fis (@) 1051 (b) 1105 (EE Adv. 2014) @ Bb @ 120 BJ MCQs with One or More than One Correct IFC, stands for "Cy, then the sum of the series 2CP 43 ot DM DET, ‘where m isan even positive integer, is equal to (1986-2 Marks) EBD_7202 ST @ 0 © Cyn © Cy" n+2) @ CI" (©) none ofthese 1 F acon Eizpten Bae ets 998-2 mary @ (Da, ©) na, al © 50, 5 BOW 11979) 42m Cy! n(x! r=0,1,2, 2n (@)_ None of the above Given that C4234 362+ ent whee Ona Prove that CP-263+3C2- = 2nCy?=(- 1G, Prove that 7+ (2 3)3°~) is divisible by 25 for'any natural number n (1982 -§ Marks) T+) = Cyt Cyr C+ + Ca then show tha the sum of the products of the C;s taken two at a time, eon representedby LGC; is ual 02?! 2 x ostejen (1983 -3 Marks) Use mathematical Induction to prove : If mis any odd positive integer, then n(n?~1) is divisible by 24 (1983 -2 Marks) fp be a natural number then prove that p"*! (p++ 1)! is divisible by p?+p +I for every positive integer (1984-4 Marks) Given =legtg oth (2) 2G oa (EY) ae trove iat Sy HG 4G 4 NGS +4 Cys, <2"S, (1984-4 Marks) Use method of mathematical induction 2.7" 3.5"—$ is divisible by 24 for all x>0 (2985 § Marks) Proveby mathematical induction that~ (1987-3 Marks) m4 2m? ~ Gast? Let Re (55-4102 and f= R= (A), where J denotes nea forall positive Integers n the greatest integer function. Prove that RY (1988 - § Marks) 10, Using mathematical induction, prove that(1989 -3 Marks) MONCp4™C "C4 + MOG = MC, where m,n, kare positive integers, and ?C, =0 forp2,where C, =" so an on c 2n_ ig an integer for eves 3 105 ” positive integer m (1990 2. Marks) Using induction or otherwise, prove that for any non- negative integers m,n, rand k, (1991 4 Marks) -micsmh ookenf nt] m! Lest #2. Pa 2 wt Sate-2¥ = ¥ p,¢e-3)" and a, = 1 for all as a > mthen show that b, = 2"*'C,,, (1992 6 Marks) Letp > 3 bean integer and a, be the rots of x°—(p+ I+ 1 =O using mathematical induction show that a" +p" © isanintegerand (isnot divisible by p (1992 6 Marks) Using mathematical induction, prove that tan“'(1/3)+tan"(1/7) +... tan“ Ln? +41 = tan (nin 2)} (1993 - 5 Marks) A Prove that (-3)"! "Cap-1 = 0, where &=(3n)/2and ‘nis an even positive integer. (1993 - 5 Marks) Ifxisnotan integral multiple of2x use mathematical induction toprove that (1994-4 Marks) nel come 460822 +... Cosmet = C05 xsin™ cosee = cox +008: osm = c08 "xsin S cosee Let mbe a pesitive integer and (EP asta SE ont yy Show that a?—a2+a2 +a,2=4, Using mathematical induction prove that for every integer n> 1,1) isdivisible by 2°" but not by 2" (1996 - 3 Marks) |, 2 4m. Use mathematical induction to (1994-5 Marks) Let 0<4, 1 isanatural number. {You may use the fact that psinx+(I-p) siny < sin [px + (I-p)y], whereO 0 and let ,=¢. Prove by induction that is well ~ defined and (a, +a, +. a, foralln=1,2, (Here, ‘well-defined means 6. Section-B J) 33 CIWAGIS53 ‘The coefficients of » and 2 in the expansion of (1+x)P"* are 12002] (@) equal (©) equal with opposite signs (©). reciprocals ofeach other (@_ none of these [Ifthe sum of the coefficients in the expansion of (a + b)*is| +4096, then the greatest coeficient in the expansion is [2002] @ 154) M2 ©) MA) MH ‘Thepositive integer just greater than (1 +0,0001)!0® jg 12002] @4 ws ©2 ‘rand n are positive integers r> 1, n> (#2) term and 37 term in the expansion of (+2) are equal, then equals 12002] @ % it © % — @ 3H 160,= [7+ Teftor bang n radical signs then by methods of mathematical induction which is true {2002[ @) a>7¥n21——@) ay SK +1) © S(K)> S{K+1) (@ S() is correct “The coefficient of the middle term in the binomial expansion inpowers of xof (I+-ax)* and of (1 ox)° is the same if. equals [2004] 3 0 @F OF © Rory 10. 2. 2B. 14, 16. n. ee ropic-wise Sotvedt Papers - MATHEMATICS “The coeflicient ofxin expansion of(1 +) (I~) is 2 4 @ Cyn © Carn) (2004) 19. Statement as Ber + "Cy = ln 12008) © Mew eas Satement-2: (+0) "G2 a ¢.sayt tmm(ltayt! The value of °c, + 5° 5°"C, is [2005] (2). Statement Ii false, Statement-2 is true z= (b).Statement-1is tue, Statement2istru; Statement-2is @ 4 HS © MG @ MC, ‘correc explanation for Satement- 19 ° (©) Statement is true, Statement-2is true; Statement-2 aL} {}anar=[f {] ten wbichoncorte tong isnot acorrct explanation fr Statement (qd) Statement -1 is true, Statement-2 is false holds for all n = 1, bythe principle of mathematical induction 20, ‘The remainder left out when 82" (62)""" is divided by 9 12005)" is, 12009] (@) AM =nd-(m-I)L) AP = 2" A-(n-I @2 &7 © 8 @o nent ol Asie ey 8 0 © grammy) A= MMA 4 Le 5,2 Ls 0I%Cy. 5 = FMC and 5, «8.2%, Te he coetisient of 27 in [ax«(,L)]” caus the wt -7 iy" (2010) cooicient of af. (a) then and b satisfy (@)--Statement-1is tru, Siatement-2itruc; Statement-2 vb! ‘is nota correct explanation for Statement -1. on pay 20081 (&)_‘Statement-1 is true, Statement -2is false ia © stb- (©). Statement-1 is alse, Statement 2s true © @ aber (@) Statement | is true, Statement 2is rue; Statement-2 isa correct explanation for Statement - If-x is so small that xand higher powers of x may be 22, ‘The coefficient of x” in the expansion of (I—x—x2-+x° fis 3 ; on) j euinn 7435) se sprouted @ 4232) 14 © 1 @ neglected, then" 2") maybe approximated as " n a ” 23. Ifmisa positive integer then (V3+1)""-(V3—1)"" is (@)_anierational mber owl 22 4r00s} (b) an odd positive integer ® @ 3x43 [2005] (©) aneven positive integer AA (@)_arational number other than positive integers © @ Pe 24. The term independent of x in expansion of If the expansion in powers of x of the function a y° Caer 1 PIT re is ay + 0,4 0x7 +4527... then a, is @ 4 &) 120 (©) 210 (@ 310, (Haye) 25. If the coefficents of x° and x in the expansion of w & 12006) (1+ax+éx")(1-2x)" in powers of x are both zero, then o-a oe : (@,b)is equal to: VEE M2014) anor pet M2). (¢28 a asi © @ 2s ose et w (+2) (422) (H88} a8) ‘or natural numbers m, nif (1 =y)"(1 +9) 26, ‘Thesum of coefficients of integral power ofx in the binomial =1+ay tay’ +... anda, =@,=10, then (m,n) is “sum i nnn ‘mia @ bss) (0)°530) {2006} expansion (1-20) eest2n) © 4535) (@) 65.45) , 1 Inthe binomial expansion of(a—6),n 25,thesumors® —@ (3-1) © 4°41) and 6" terms iszero, then ab equals 12007] ; ; ne 6 (@) >(3°+1) @) 5(3*) 3 © os © 26°+)) @ 30") ‘The sum ofthe series 120071 27, ifthe number of erms in the expansion of (1-244, 265 96,4 96, = C4. maart Ey is ¥ 1 0 & mG, © Go @ 3°Go x0, i828, then the sum of the coefficients ofall the terms inthis expansion, is JEM 2016] @ 2B) M ©) H — @ 2187 GP_3480 CHAPTER Sequences and Series [J Section-A MFFT ESTs 4’ BE TThesum of integers from Ito 100 that are divisible by2 or S is (1984-2 Marks) Thesolution ofthe equation logy logy(Vr=3 + Vi) =Ois (1986 ~ 2 Marks) Feo Gum of the first n terms of the. series 1742.27 437 42.47 457 42.67 +, is n (+1)? /2, when nis even, When m is odd, the sum is (1988 - 2 Marks) iethchamonicmean and gometriemen oftwo postive tmbesbe heron 4:5. Then the two suber inthe no (1993-2 Marks) Forany odd integern > 1, n°4n-I)*..4(-1)"! (1986-1 Mar) Let p and g be roots of the equation x*~2x + A= Oand let r and s be the roots of the equation x? ~ 18 + B = 0. If pegcrdsacinantmccprogension ben tod (1997-2 harks) c EIR Ifx,yand.zarepth, gth and rth terms respectively ofan A.P, and also of a GP, then w~£y*-*2*Y is equal to @ yb) 0 © 1 @ Noneofthese ‘Thetthird term of a geometric progression is 4. The product ofthe first five terms is (1982-2 Marks) @ 8) 4] 4 G@ none of these ‘Therational number, which equals the number 2.357 with recurring decimal is (1983-1 Mark) (@ none of these ao ® or jor © Ifa, b, care 999 then the equations ax? +2br-+ and de? +2ex+ f =0 haveacommon rootit £5 are (1988-3 sharks) @ AR GE © HP — @ noneofthese Summ ofthe ist n terms ofthe series 1,3 1,3 ‘sequal to (1988-2 Marks) 2°4°8 "16 and a @ Wnt @ 1-2" (@ ner" @ 241. 6 n. 2 1B. 16. ‘Thenumber log 7 is (@). aninteger (b) rational number (©) anirrational number (@)_aprime number Ifln(a +c), In(a—c), In(a-2b+ are in A.P, then (1994) (@) a,b,carein AP, ©) 2,6, Parein AP. (©) a,b,careinGP. (@) a,b,carein HP. (1990 2 Marks) Oe eee aA Dank heh bene HELE 2 ee ree eRe rly @) 2 (bo) 3 @ 5 @ 6 The harmonic mean of the roots of the equation (5+) x2 (4048) x84 245 «018 1999-2 Mars) ®2 4 ©6 @8 Consider an nite gomstric eis with ist tem a and common ratio Its sum is 4 andthe second term 31, then 200s) 3 Oy ) a=2 © @ an3r4 Leta, Bethe roots ofx2—x+ p= Oandy, te the roots of de q=0 Ifa .7.SareinGP, thentheincgral ales of pand respectively are (2m) @) 220) 23 @ 63 (6-2 Letthpostivenumbers a,b,c, bein A P.Then abe, ab, acd, bed are (20018) (@) NOTMAP/GP/MP (6) inAP (©) inGP @) inte Ifthe sum ofthe first 2n terms ofthe A.P.2, 5,8, ... is equal tothe sum ofthe firstm terms ofthe A P. 57, $9, 61, then n equals (20018) @ 0 OR ©, @B Suppose a, b, © are in AP. and a2, 6%, c? are in GP. if 3 «then the value of is (20025) hood L 1 © TF OT © FE OE ‘An infinite GP has first term ‘x’and sums’ then x belongs a10 Inthe quadratic equation ax?+bx-+c=0,4 =82 —4acand +B, a2+B?,a° +, arein GP where, arethe root of ax?+bx-+e=0, then 20055) (@ A20 (b) DATO @ cA=0 (@) A=0 EBD_7202 Se) 07 17. ta cue sum of first m terms ofan A P.is en? then the sum of squares of these m terms is 2009) w@ MG =Ne? gy nde? sie? @ ae tw “ers (dn? — tye? nae @ ae oS 18. Let ay, ay, ay... bein harmonic progression with a, =S and dig ~ 25. The least positive integer » for which a,<08 eon) @2 HB @ mM ws 1. 2, 101, Suppose og, b,, 10g, 03, big, ate in Arithmetic Progression (A.P.) with the common difference log, 2. Suppose a, a. are in A.P such that and a, by. Ift=b,+b, +--+ By and s=a,-as + a}, then (EE Ads. 2016) (@)" stand a> be, (b) 5> Land ay, by, — (@) s1,y>1,2> 1 arein GR, then Ty ng’ Teln yen S (1998-2 Marks) (@ None of these arein @ AR) HR (@ GP For a postive integer n, let ra 4 a(m=1+ 9434 g te ay Ten (1999-3 Marks) @ (100) < 100 (©) @(100)> 100 (©) 40200) < 100 (@ @(200)> 100 1 -awise Solved Papers - MATHEMATICS AA straight line through the vertex P of a triangle POR intersets the side QR atthe point Sand the cireumeirele of the triangle PQR atthe point T.IfS isnot the centre ofthe cireumeitele, then (2008) 2 SxS Tete © m<3R kas) (1) * 2 Then S, can take value(s) (JEE Adv. 2013) @) 1055) 08) IM BD 5 EOS ‘The harmonic mean of two numbersis 4, Their arithmetic mean 4 and the geometric mean G satisfy the relation, 2A+G2=27 Find the two numbers. 4979) The interior angles of a polygon are in arithmetic progression, The smallest angle is 120", and the common difference is 5°, Find the numberof sides ofthe polygon (1980) Does there exist a geometric progression containing 27, 8 and 12 as three of its terms 7 IFit exits, how many such progressions are possible ? (1982 -3 Marks) Find three numbers a, 6 ¢, between 2 and 18 such that (their sum is25 (the numbers 2, a,b sare consecutive terms ofan AP and (ii) thenumbers bc, 18 are consecutive terms ofa GP. (1983 -2 Marks) MWa>0,b>Oande>0, provethat Ltd @+b+o[r+pei]29 (1984-2 Marks) fn isa natural number such that PADS PSS cooepAM aNd Phy Pay ns Py AE distinct primes, then show that Inn > kn2 (1984-2 Marks) Fin beounet bess tne LT Ae oy tae Ta tom ms Sewer Z Eww mem) (2985-5 Marks) GP_3480 Sequences and Series 8 10, 1B. 4 18, 16. Solve for the following equation: (1987-3 Marks) Togans(62 + 238421) = 4-loggayry (te? +1289) Wee2 by 29a "2 ae progression, determine the value of x. (1990-4 Marks) Let p be the first ofthe m arithmetic means between two ‘numbers and g the first of m harmonic means between the same numbers. Show that q does not lie between p and. 2 a (1991-4 Marks) Ws, _ 5, ate thesums finite geometric setes whose fist terms ate 1,2, 3, MANE Whose in 1 common ratcsare 1.11, 1 respectively sa ntl * then ind the vals of 2 +52 +82 sat SB (1991-4 Marks) ‘The real numbers x), x), x; satisfying the equation 2° — x7 +8x+>)= Oarein AP. Find the intervals in which Band lie, (1996-3 Marks) Leta, 6 ¢, d be real numbers in GP. If, v, w, satisfy the system of equations (1999-10 Marks) then show that the roots ofthe equation Pete tle F1b-eF + (ea) +(d-B)] x+utvtw=0 and 20x + 10 (a- d)?x-9 = Oare reciprocals of each other. Te fourth power ofthe common diference ofan arithmatic progression with integer entries is added tothe product of any four consecutive terms of i. Prove thatthe resulting sum isthe square ofan integer. (2000-4 Marks) Leta), a>, -, dy be postive real numbers in geometric progression. For €ach n, let dy, Gy, H, be respectively, the arithmetic mean, geometric mean, and harmonic mean of 14, Find an expression forthe eometrie mean of 1 Gy imterms of 4,9 ys yy Hy Hy (2001-5 Marks) Leta, b be positive real numbers. If. 4y, Ay, b are avthmetic progeession, a, G;, Gy, 8 ate in geometric progression and, ,, Hy, bate in harmonic progression, GG, _ At _ Qa+byXa+26) show that 7H, Hy +H Sab (2002 - § Marks) 17. Ifa, b,c arein AP, a2, b, care in HLP, then prove that cithera=b=cora,b,—< formaGP. (2003-4 Marks) 2 2 157 . 3_(3P (3 (3 — [5] +[F] econ] an 4 (i (a) + om (i “ 4 then find the least natural nummer my such that (2006- 6M) 18 Ita, 4 5, > ay Vn > Mp G FSS PASSAGE-1 Let F, denote the sum of first terms ofan arithmetic progression, (AB) whose fist term is rand the common difference is(2r— 1), Let T,= V,,,-V,-2andQ,= 7, ,,~T,forr= 1, 2, 1. ThesumV, FV) +..#Vy (2007-4 marks) 0) Jensen? nad) anette? ene @ n™ 1X3 1) ©) mb 3 2 © Inat-ney © 40° -2003 2 Tisha (2007 -t mari) (a) anodd number (b) aneven number (© aprimenumber (4). acomposit number 3. Which one ofthe following is a cores statement ? (2007 -4 marks) (@) Qj,Qy,Q,,..areinA.P. with common difference 5 (©) 01.0s.05,...areinA.P. with common diference 6 (©) Q1.0y.Q..arein AP. with common difference 11 @ "=O," PASSAGE-2 Let Ay, G,, H; denote the arithmetic, geometric and harmonic ‘means, respectively of two distinct positive numbers. For 22, Let 4, and H, have arithmetic, geometric and harmonic sean’ as Ay, Gy. H, respectively. 4. Which one of the following statements i correct? (@) 6,>6,>6,> (2007-4 marks) 0) 6, <6,G,> G.> ‘Which one ofthe following statements i correct ? (@) A\>Ay>Ay> (2007-4 marks) 0) 4, Ay>Ag> sand Ay Ay>dq> 6. Which one of the following statements i correct? (@) Hy>Hy> Hy> (2007-4 marks) ©) WHy> Hs> @) Hy H,>H,> Ld top ic-wise Solved Papers - MATHEMATICS (Seeks ‘Suppose four distinct positive numbers a, a, ay, a4 are in GP Let =a), by=b, +a, b,=by ras and by~ by ay STATEMENT -1 : Thenumbers by, b,b;,bgareneither in AB.nor in GP. and STATEMENT-2: The numbers bb, b3,, arein HP. 2008) (@)_ STATEMENT - 1 is True, STATEMENT - 2 is True; STATEMENT - 2 is a correct explanation for ‘STATEMENT- 1 (b) STATEMENT - | is True, STATEMENT - 2 is True; STATEMENT - 2 is NOT a correct explanation for ‘STATEMENT: 1 (©) STATEMENT-1 is True, STATEMENT -2is False (@_ STATEMENT-1 is False, STATEMENT -2 is True BM integer Value Correct Type Let $,, = 1,2, ..., 100, denote the sum ofthe infinite rons vhs fentemis!=} ato conmon 1 100? Se cats tan thevaco OM « S238 ok TRanbealiea y+ DI? 3k 94 | is 2010) Let a). 5... yy be teal numbers satisfying aj°15,27-2a,>Oanday=2a. Ay OK =3,4 yo atad+....tal ig 18 iL=99, then the value of nr BAe tn FAH i equal to e010) 3. 1 Let ay, dy, dy --dyqg be an arithmetic progression with Band S,= Say < p 100, For any integer n with i 1sn<20, let m Sn 1 SE does not depend on, then ayis eon {A pack contains n cards numbered from 1 10 n, Two ‘consecutive numbered card are removed from the pack and thesum ofthemumberson the remaining cardi 1224 the smaller of the numbers on the removed cards is k then k-201 (EE Ads. 2013) Leta, bebe positive integers such that ® isan integer. IF, +b, carein geometric progression and the arithmetic mean of| a,b, cisb+2, then thevatueor ta 14 is (EE Adv. 2014) ‘Suppose that all the terms of an arithmetic progression (A.P.) arenatural number, Ifthe ai ofthe sum ofthe ist seven terms tothe sum ofthe fist cleven terms i 6:11 and the seventh term iesin btwoen 130 and 140, then the common diffrence ofthis APs (GEE Adv. 2015) Thecoecentof? i the expansion of(1+3) (1 +243) (x!®)is (EE Ads. 2015) GP_3480 Sequences and Series IJ Section-6 STR T3 & 10, IF], log, (3+ 2), log, (4.3°— 1) arein AP. then x equals 12002] @ l0g,4 (0) 1-log,4 (© 1-log,3 (@) log,3 Lem, mare the p,q! and #* term of a G Pall positive, log! p ii then log mq 1) equals 12002] logn rl @ 1 &2 o1 @ 0 Thevalucof2'4.418,81"6 sais 2002] @1 2 © 32 @4 Fifth term of GP s2, then the produc ofits 9 terms is 12002] (@ 26 @) 52 (© 1024 (@)_ none of these ‘Sum ofinfinite number of erm of GPs 20 and sum oftheir square is 100, The common ratio of GPis 12002] @ 5 & HS © 85 @ us BoB+34 +, 49 = 12002] @ & b - | mS 45 ‘The sum of the series 12003) upto oo is equal to (b) 2log,2 (@) loge 2 then 7 is equal to wo) dnt 2004 © 3 12004) @ 3" Let 7, be the rth term of an A.P. whose first term is a and common difference is d. If for some positive integers then a—d equals [2004], 1 am «(OO The sum of the first n terms ofthe series 1422743? 424? 457 426+ 2 is M0" hen mis even, When 1 is odd the sum is 2 ns), i 2004] 3n(n+1) 2 is 12004] © Ifthe coefficients ofrth, (r+ 1th, and (-+2)th terms in the the binomial expansion of (+ y)" are in AP, then mand. r satisfy the equation 12005] @) nm -mar-1) +4? -2=0 () nm? -m(dr+1)4? =2=0 (©) nm? -m(r+1)+4r? -2=0 (@ m?-m(ar-1)+47? +2=0 Hee Da" y= D0, wherea, b,carein AP mo ao andja|<1,||<1,le[(K9-(k-1 =n for any natural a number n (2012) 24. 26, a. 28, 29, 30, (@)_Statement-1is false, Statement.2 is true (©) Statement-1is true, statement-2s true; statement-2is| ‘correct explanation for Statement-1 (©). Statement-1is true statement-s true; statement-2is| ‘nota correct explanation for Statement-1 (@)_Statement-1is true, statement-2is false. ‘The sum of ist 20 terms ofthe sequence 0:7, 0.77,0.777, is EE M2013} 7 30 269-10" G@) G079-10) ey) _F99-10) 1 apos0™ 160410 © Zom+10™) — @ Fo9+10™) 16 (10 +2(19) (107) +301 (10) +... +10(11) =(10)", then kis equal to: JEEM 2014] 121 4a @ 1% wn © HF we ‘Three postive numbers form an increasing G P Ifthe midale ‘erm inthis GP is doubled, the new numbersarein A P then the common ratio of the GP is EE M2014) @ 2-3 &) 248 © SRV @ 3+v2 The sum of first terms ofthe series POP+? Paves Th as EE M 2015}, @2 wm ©7 @ % Ifm isthe A.M. oftwo distinet real numbers /and n(,n> 1) and Gy, G, and G, are three geometric means between / and, n,then Gf +2G$+Gfequals [JEE M2015) (@) 4 mn? (b) 4Pm?n? (© 42mn (0) Aten Ifthe 2%, sand 9 terms ofanon-constant A.P.areinG.P., then the common ratio of this G.P. is SEE M 2016] 1 @1 oF 8 4 Og 5 If the sum of the first ten terms of the series (J +03) (4) ei(ed} ten ie!Se, 3) 115) "Bs 3 5 then misequal 0 eEM 2016 (@) 100 (©) U2 ©) ® @ 10 CHAPTER Straight Lines and Pair of Straight Lines If Section-A 4’ BE ‘Thearca enclosed within thecurve|x|+|y1= lis. (1981-2 Marks) y= 10" isthe reflection of y = logyg x in the line whose equations (1982-2 Marks) The set of lines ax-rby+e = 0, where 3a + 2b + 4c = O is concurrent atthe point (1982-2 Marks) 4. Given the points 4 (0, 4) and B (0,4, the equation ofthe locus ofthe point Px, y) such that | AP-BP|=6is. (1983-1 Mark) 5. Ifa, band carcin AP, then thestraight lineax-+by~c=0 will always pass through a fixed point whose coordinates are (1984-2 Marks) 6, The orthocentre ofthe triangle formed by the lines xty= I, 2e+ 3y= 6 and dx— y+ 4 = 0 lies in quadrant number (1985-2 Marks) Let the algebraic sum of the perpendicular distances from the points (2,0), (0,2)and (1,1) toa variable straight line be zero, then the line passes through a fixed point whose cordinatesare (1991 2.Marks) ‘The vertices ofa triangle are A(-1,~7),B(S, 1) and C(I, 4). The equation of the bisector of the angle Z4BCis| (1993- 2 Marks) ’ Rr ‘The straight line 5x + 4y = 0 passes through the point of intersection of the straight lines x + 2y ~ 10 = 0 and, dee y+5=0, (1983-1 Mark) ‘The'lines 2x + 3y + 19 = 0 and 9x + 6y — 17 = 0 cut the. coordinate axes in concyelic points. (1988 I Mark) c EIReiireirremarues ‘The points (~a,~6),(0, 0), (a, 6) and (¢*, ab) are (@) Collinear (©) Vertces of parallelogram (©) Vertices ofa rectangle (None of these The point (4, 1) undergoes the following three transformations successively. (1980) (Reflection about the line y= x. (Gi) Translation through a distance 2 unitsalong the postive direction of x-axis, (ii). Rotation through an angle p/4 about the origin in the ‘counter clockwise direction, (1979) ‘Then the final position of the point is given by the coordinates. 1a R 4 (bo) (-v2,7v2 4) &) ) 1 © (ee) “Thestraightliness+ y= triangle which is @ (2,742) ) 3+ y—4 = 0.04394 =0 form (1983-1 Mark) (@) isosceles (b) equilateral (©) rightangled (@) none of these IfP=(1, 0), O=(-1, 0)and R= (2, 0)are three given points, then locus of the point satisfying the relation SO? + SR?=2SP, is (1988 - 2 Marks) (@)_ astraight line parallel tox-axis (b) acircle passing through the origin (6) acitele withthe centreat the origin (@)_astraigth line parallel toy-axis, Line L has interceptsa and b onthe coordinate axes. When theaxes arerotated through a given angle, keeping the origin fixed, the same line J has intercepts p and q, then (1990 2 Marks) @) aoe? = preg? © abs pr awe? P 9 ‘Ifthe sum ofthe distances ofa point from two perpendicular lines ina planes 1, then itslocusis (1992- 2 Marks) @)_ square (b) circle (©) straight ine (@)_ two intersecting lines ‘The locus ofa variable point whose distance from (-2, 0) is 2/3 times its distance from the line x: is 1994) (@) llipse (b) parabola (©)_ hyperbola (@)_ none of these “The equations to a pair of opposite sides of parallelogram are x2 Sx-+ 6= O.and y? 6) +5=0, the equations to its diagonals are (1994) (@) x+4y=13,y=4e-7 (b) Art y=13,4y=x-7 (©) 4r+y=13,y=4r-7 @) y~4r= 13, yar: The orthocentre of the andx+y BI angle formed by the lines xy = 0 (19958) ooo (2) EBD_7202

Você também pode gostar